1

7th & 8th March-2016- Papers of all Specialties (1705 MCQS)

[ Index- Check List ] Compiled by : Amlodipine Besylate

(1) Medicine & Allied 7th March 2016 (Evening Session) by Alizay Khan (181 MCQS) Page#1 (2) Medice & Allied 8th March 2016 (Morning Session) by Dr Kunza Aslam (200 MCQS) P#15 (3) Medicine 8th March(Evening) by Dr.Tariq Khan/Mudassir Bangash (200MCQS) P#29 (4).Surgery & Allied 7th March (Evening Session) by Dr. Hasnain Afzal (197 MCQS) P#40 (5). Surgery & Allied 7th March (Evening Session) - by Dr.Xaheer Khan (185 MCQS) P#57 (6). Surgery 7th March 2016 (Morning Session) by By Dr.Haris Riaz Sheikh (156+) P#73 (7). Gyane/Obs 7th March-2016 (Morning Session) by Dr.Noor Fatima (184) P#89 (8)..Gynae / Obs; 8th March 2016 (Morning Session) Dr.Nourin Hameed (105) P#94 (9). Radiology 7th March-2016(Morning) by Dr.Asfandyar Khan Bhittani & Loa Loa(122) P#103 (10). Community Medicine 7th March 2016 (Morning) by Dr.Qaisar Javed (90+85) P#112 =-=-=-=-=-=-=-=-=-=-=-=-=-=-=-=-=-=-=-=-=-=-=-=-=-=-=-=-=-=-=-=-=-=-=-=-=-=-=-=-=-=-=-=-=-=

(1)Medicine & Allied 7th March 2016(evening) by Alizay Khan (181) 1. anterior cruciate ligament is damaged.direction of tibial dislocation on femur is a. anteriolateral b. anteromeddiaal c. anterior (answer) d. posterromedial e. posterolateral 2. narrowest point in pediatric airway a. cricoid (answer) b. thyroid c. trachea d. false vocal cord e. true vocal cords 3. regarding vertebral column a. intervertebral disc is thickest in thoracic and lumber regions b. cervical vertebrae are 7(answer) c. total 31 vertebrae d. curvature to side is caalled lordosis e. prolapse can occur without fracutre 4. pulmonary supplies a. alveoli (answer) 5. if thalamus gets damaged,which is not affected a. sense of olfacton (answer) b. vibration c. touch 6. dorsal nucleus of vagus nerve is present a. upper part of pons b. lower part of pons c. medulla (answer) d. midbrain 7. superior colliculus recieves which sensation a. auditory b. visual (answer) c. hearing 2

8. posterior to urinary bladder a. anal canal b. prostate c. denonvillers fascia (answer) d. completely covered by peritoneum e. covered by peritoneum in lower half only 9. dorsal column damaged,which will be least affected a. touch b. sterogonosis (answer-NOT SURE) (temp and vibration were not in options) 10. nerve involved in bell's palsy a. 7 (answer) b. 9 c. 10 11. posterior communicating artery a. passes below the occulomotor nerve and connects ICA and PCA (answer) b. passes above occulomotor nerve and connects ICA to MCA c. connects ICA to posterior cerebellar rtery 12. structure passing thru the cavernous sinus a. CN 3 b. CN 6 (answer) c. CN 4 13. isthumus of thyroid is situated over which tracheal rings a. 1,2,3 b. 2,3,4 (answer) c. 1,2 d. 2,3 14. which muscle gets paralyzed/damaged in temporomandibular joint dislocation a. temporalis b. masseter c. lateral pterygoid (answer) d. medial pterygoid e. buccinator 15. fascia deep to parotid gland forms a. stylomandibular ligament b. stylohyoid ligament c. temporomandibular ligament (answer) 16. cardiac plexus a. formed infront of trachea b. have preganglionic sympthetic fibers from superior cervical ganglia c. contain both sympathetic andd parasympathetic fibers (answer) 17. 1st branch of abdominal a. inferior phrenic (answer) b. celiac 18. LCX blocked.area affected a. left atrium+left ventricle (answer) 19. brachial plexus block...pt still feels pain on lateral aspect of forearm.which nerve is spared a. axillary nerrve b. musculocutaneous nerve (answer) 20. structure pierced during spinal tap 3

a. posteerrior longitudinal lig b. ligamentum flavum (answer) 21. dilatation of aortic arch will compress a. left bronchus (answer) b. trachea c. esophagus (RLN was not in options) 22. muscles are connected to bones via a. ligaments b. tendons (answer) 23. long thoracic nerve supplies a. lattisimus dorsi b. serratus anterior (answer) 24. pt with spells of dizziness and diplopia,now presented with coma.what's the diagnosis a. subarachnoid hemorrhage b. carotid arterry obstruction c. basilar artery thrombosis (answer) d. pontin hemorrhage 25. axillary artery is formedd by a. bassilic and vena commitantes of brachial artery (answer) b. cephalic vein and vena commitantes of brachial artery 26. at which level trachea starts a. c6 (answer) b. t? 27. jejunum is supplied by a. SMA (answer) b. IMA c. pancreaticoduodenal artery 28. regarding vessels of lower limb a. cruciate anastmosis is b/w femoral artery and external iliac artery b. short saphenous starts from medial side of foot c. long saphenous ends in mid thigh d. femoral vein is lateral to femoral canal (answer) 29. injury to lateral hypothalamus will a. increases appetite b. no effect on hunger c. decreases hunger (answer) d. increases hunger for carbohydrates 30. lesion of rt lateral geniculaate body causes a. left homonymus hemianopia (answer) b. rt homonymus hemianopia c. bitemporal hemianopia 31. internal carotid artery a. ophthalmic branch enters thru superior orbital fissure b. it enters thrusquamous paart of temporal bone c. at bifurcation its lateral to external carotid artery then turns medially and posteriorly (answer) d. pierces diaphragma sella medial to optic nerve 32. bundle of his is supplied by a. RCA b. LCX 4

c. rt marginal artery (answer) 33. lymphatic drainage of thyroid a. deep cervical nodes (answer) 34. in open laparotomy,surgeon recognizes left kidney by a. dark color b. begins from T12 to L4 c. anterior relation with stomach,pancrease,descending colon,spleen,jejunum (answer) d. anterior relation with pancrease,descending colon,spleen,jejunum 35. pt suffering from appendicitis,having pain in right iliac fossa that is reffered to umbilicus.which spinal nerve root is involved in this reffered pain a. T10 (answer) b. L5 36. light thrown in rt eye,direct reflex is present but absence of indirect light reflex.which structure is involved/damaged a. rt occulomotor nerve b. left occulomotor nerve (answer) c. pretectum 37. linear growth pf bone is affected if following structure is fractured a. metaphysis b. epiphyseal line c. epiphyseal plate (answer) d. diaphysis 38. abscess anteior to pretracheal fascia.infection can spread to a. anterior mediastinum (answer) 39. beta endorphins a. hypothalamus 40. typical scenario of sickel cell anemia with abdominal pain jaundice etc... a. HB ss (answer) b. HB sc 41. difference b/w sarcoma and carcinoma a. pleomorphism b. inc vascularity 42. ITP scenario was given.splenectomy done.which organism will cause postsplenectomy reecurrent infections a. hemophilis influenza b. staph aureus (answer) 43. hepatitis scenario,viral markers negative,ALT slightly raised,on exaamination there were some eye findings.what is the suitable dignostic test a. serum ceruloplasmin (answer) b. urinary copper c. HbsAg d. anti mitochondrial ab 44. patient comes with tender right hypochondrium,stool with occult .on colonoscopy, ascending colon was involved, showing ulcers while other parts of colon were spared.on histology/microscopy (smthng like that)what will be seen a. necrotizing vasculitis b. crypt abscess c. entamoeba histolytica d. carcinoma 45. verrrrrryyyy long stem.microscopic findings were given.2 word thats i remember from those alien features are epithelium and rete ridges 5

a. verrocus carcinoma b. squamous cell carcinoma(or small cell) these two were carcinomas. remaining were sarcoma etc 46. child with history of bed wetting though toilet trained.investigations done.at one side duplication of ureter was found and one of them was opening in vagina.what is the cause a. early division of ureteric bud 47. coal minning,sand blaster.lon|Achg scenario a. asbestosis b. silicosis (answer) d. anthracosis 48. anxiety is dec by the activation of which receptors a. GABAa b. glutamate c. glucocorticoid d. nicotinic cholinergic e. dopamine 49. turner syndrome a. AR b. AD c. gynaecomastiaa d. short stature (answer) 50. CRF pt died.on autopsy,which organ will show hypertrophy a. thyroid b. parathyroid(answer) 51. which cell organelle contain double membrane a. nucleolus b. golgi apparatus c. ribosomes d. RER (answer-??) 52. fisherman with gingival hyperplasia and echymosis.which one is deficcient a. vit B12 b. vit K c. vit c (answer) 53. pt on immunosuppresents.abscess is formed on upper outerr half of arm.after drainage healing process is verry slow though a month hs been passed.reason a. dec collagen formation b. dec neutophil migration 54. slowest growing malignant thyroid Carcinoma a. papillary (answer) b. follicular 55. which infusion will inc ECF to maximum extent a. hypertonic fluid (answer) b. isotonic fluid 56. steady pressure is detected by a. meissners b. ruffinis (answer) c. pacinian 57. pt with MI.typical sign and symptoms.st elevation in leads II, III and AVF a. anterior wall MI b. inferior wall MI ans: b 6

58. pt with AML undergone bonemarrow transplant....pneumonia...histology showing large cells with intrcellular inclusions.cause is a. CMV b. candida c. pneumocystic ans: a I marked 59. child brought by mother,suffering from diarrhea.on examination,rectal prolapsed was found(finaly a hint was given that worm with small anterior end) A. entrobius vermicularis b. trichuria trichuris c. whip worm ans: b 60. best indicator of venous return a. end diastolic vol b. end systolic vol ans: a 61. MI pt after 48 hour,found collapsed,no pulse.on ECG irregular waves were seen.whats the pathophysiology a. dec cardiac output b. re-entery current ans: I marked b 62. subcapsular efferent a. lymph nodes b. spleen c. thymus ans: a 63. alveoli are kept dry because of a. alveolar macrophages b. tight junction b/w c. surfactants d. negative interstitial pressure 64. dead space does not change in a. shallow breathing b. deep inspiration c. standing ans: a 65. local edema a. allergy 66. dec GFR a. inc pressure in bowman's capsule b. dec hydrostatic pressure c. inc oncotic pressure ans: a 67. what is more/high in venous blood as compare to arterial blood a. PCV 68. long senario,child with hypocalcemia(or tetany),having some cardiaac problems and lots of viral infections mentioned. a. NADPH oxidase deficiency b. 22q- c. lowCD4 ans: b 7

69. cause of fatty liver in our country a. hep B and hep C b. alcoholism c. fatty diet ans: a 70. primary active transport a. pump b. carriers c. channel proteins ans: a 71. corona radiata is formed by a. granulosa cell b. theca interna cells c. theca externa cells ans: a 72. spermiogenesis a. primary oocytes form secondary oocytes b. secondary oocytes form sperrmatids c. spermatidz form spermatozoa ans: c 73. renal threshold for glucose a. 180 b. 200 c. 250 d. 375 ans: I marked c 74. in which condition atrial repolarization is shown on ECG a. third degree cmplt block b. inspiration c. during exercise ans: a 75. on ECG,QRS complex represents a. ventricular systole b. ventricular depolarization ans: b 76. non smoker,fever for 6 weeks and cough with small amount of blood.on xray "Coin lesion"is seen in upperr lobe a. small cell carcinoma b. granulomaa c. silicosis 77. which structure binds cytoskeleton with ECM. a. proteoglycans b. intermediate filaments c. integrins ans: a 78. a tall young girl(child) with history of secretion from breast.on microscopy of anterior pituitary,which cells ar abundant a. eosinophilic b. basophilic c. chromophobes ans: a 8

79. which valves are most commonly involved in rheumatic heart disese a. mitral and aortic 80. aseptic endocarditis a. SLE b. cancer patients ans: I marked b 81. pt with muscle weaakness and bilateral ptosis.which investigations you will do a. Acetylcholine receptors antibodies b. anti smooth muscles antibodies (EMG was not in options) ans: a 82. farmer's a. sugarcane dust b. grain dust c. cotton dust d. tobacco ans: b 83. hormone responsible for ductal growth and fat deposition in bresat a. progesterone b. estrogen c. prolactin ans: b 84. heart rate is 75,PR interval is 0.3.if heart rate becomes 225, what will be the PR interval a 0.1 b 0.9 c 0.05 ans: a 85. least amount of minerals are found in a. roots b. tubers c. cereal d. pulses e. veg(leaves) 86. erythropoietin secretion iss inhibited by a. cobalt b. hypoxia c. theophylline ans: c 87. which is low in csf as compare to plasma a. Na b. osmolarity c. Cl d. Mg e. protein ans: e 88. cells of chronic inflammation a. macrophages b. lymphocytes ans: a 89. middle aged man presented with meningitis after having lung abscess 9

a. staph aureus 90. which toxin is responsible for scarlet fever a. exotoxin b. erythrogenic c. endotoxin ans: b 91. serum sodium is regulated by a. osmoreceptors 92. highest sodium channel conc a. initial segment b. node of ranvir c. dendrites ans: b 93. in chronic liver disease,which histological finding suggests chronicity a. fibrosis b. councilmann bodies ans: a 94. FFPs used for the acute management of a. factor 8 deficiency b. warfarin overdose ans: b 95. intense transfusion reaction occurs if we transfuse a. A- to A+ b. A+ to O+ ans: b 96. which prevents muscle from tearing unde pressure a. GTO b. muscle spindle ans: a 97. inverse stretch reflex 98. flexor withdrawal reflex a. multisynaptic 99. if right atrial pressure is increased a. increases cardiac output b. increases intrathrocic pressure ans: a 100. after adrenalectomy,which is preffered a. glucose b. NaCl ans: b 101. in cell mem of RBCs,CL- and HCO3- exchange occurs through a. nkyrin b. band3 c. spectrin ans: b 102. pt with discoid rash,arthritis and hemolytic anemia.whats most specific investigation a. ANA b. ant ds DNA ab ans: b 10

103. pt with malaise and raised bp,died b/c of hemorrhagic stroke in basal ganglia.on autopsy bilateral small kidneys with petecheal hemorrhages,hyperplastic and fibrinoid necrosis.whats the diagnosis a. fibromuscular dysplasia b. DM type II c. systemic sclerosis d. NSAID induced 104. which one is most pre malignant a. compound nevus b. intradermal nevuss c. seborrhaic keratosis ans: a 105. old male patient hass history of headache in temporal region.thick cord like vessels in that area.on biopsy giant cells are seen a. ESR more than 110 weird options... 106. presynaptic sympathetic fibers release a. ACH b. nor epinephrine c. dopmine ans: a 107. saliva prevents iron utilization by microbes via a. lactoferrin b. lysozymes ans: a 108. benign tumor a. leiomyoma 109. pt with left ventricular hypertrophy(smthng like this,finaly asked)what is the cause of left axis deviation a. RBBB b. damaged myocardium of right ventricle ans: b 110. sulfur containing aminoacids a. cystine 111. which corneybacterium is common amongst humans a. diphteriae 112. diphtheria toxin has sever affect on a. heart b. brain ans: a 113. anemic pt, Hb 6.2 platelets 450,000 TLC normal range BME shows erythroid hyperplasia a. acute blood loss b. leukemia c. iron deficiency anemia d. anemia of CRF 114. hyperkalemia will increase the secretion of a. aldosterone b. ADH c. d. cortisol 11

ans: a 115. urine osmolarity 1200(or 1400)plasma sodium 120, whats the cause a. increase ADH 116. urine concentrated in juxtamedullary nephrons.in which part dilute urine will be present a. DCT b. CT c. thick ascending loop of henle d. thin ascending loop of henle ans: c 117. stab wound of chest,what will happen a. ipsilateral lung collapses and ipsilateral chest wall springs out b. ipsilateral lung collapses and contralaterl chest wall springs out ans: a 118. 46 XX pseudohermaphrodite a. adrenogenital syndrome 119. long scenario,at the end asking bout type of necrosis in brian a. coagulative b. liquefactive ans: b 120. uncoupling of oxidative phosphorylation and heat production,hormone involved a. thyroxine 121. tumor suppressor gene a. p53 b. Ras c. n.myc d. c.myc ans: a 122. young male is suffering from syphilis(i think primary mentioned).to confirm the diagnosis, smple is taken from a. blood sample b. genital sores c. buccal mucosa ans: b 123. dec heat production during anesthesia is due to(something like this) a. reduced activity of Na/K pump b. reduced skeletal muscle tone c. d. starvation??? 124. which factor helps immune system against microbial defense a. complement system b. c3b ans: I marked b 125. occupational carcinoma a. tobacco factory worker b. silicosis c. asbestosis ans: c 126. abnormalities are common in meiosis.if a complete chromosome is transferred to haploid cell(dont remember haploid word was present or not)what this abnormality is called 12

a. trisomy b. meiotic nondysjunction 127. vagotomy done a. dec gastric acid and pepsin secretion 128. commonest organism causing UTI a. ecoli b. candida ans: a 129. sweat glands a. not present in palm and sole b. innervated by sympathetic nervous system ans: b 130. Na is major ECF cation.its mostly balanced by which anion a. HCO3- b. Cl- ans: b 131. which is more in dialysing fluid as compare to plasma a. glucose b. HCO3- ans: a 132. which inhibits stomach emptying a. gastrin b. CCK ans: b 133. which causes gall bladders contraction a. CCK b. gastrin ans: a 134. dec gastric secretions a. gastrin b. secretin c. enterogasterone ans: b 135. what is glycocalyx a. structural protein b. carbohydrate moiety c. receptors... 136. sarcoma as compare to carcinoma a. spread to bones early b. in the form of clusters of cells c. more sensitive to radiotherapy 137. S2 as compared to s1 has a. high frequency b. longer duration ans: a 138. in a standing person,venous return from legs i facilitated by a. contraction of skeletal mucles b. valves in ans: a 139. gastric lymphomas a. H.pylori 13

140. T cell receptor interacts with a. antigen presenting cells b. MHC ans: b 141. median umbilical ligament is remanant of a. urachus 142. female pt in reproductive age presents with DVT,there is history of taking some medicine,whcih it could be a. OCPs 143. ch 9:22 translocation a. CML 144. dec ESR a. inc albumin 145. cells of pancrease involved in DM I a. alpha cells b. beta cells ans: b 146. primary biliary cirrhosis,typical senario of female with history of itching ,jaundice xantoma...investigation a. anti mitochondrial antibodies 147. hematocrit a. rbcs+wbcs+platelets 148. achalasia cardia dont remember options..absent gaglion or something was there which was the right option 149. 150. 2 qstns related to asthma.cant recall 151. lymphatic tumor a. cystic hygroma 152. HIV patient...tumor a. kaposi's sarcoma 153. HLA DR4 a. RA 154. scenario of ankylosing spondylitis..asked about association a. HLA-B 27 155. Antibody receptors are a. hypervariable regions of H and L chains 156. sympathetic supply to adrenal medulla (easy qstn so dont remembr) 157. negative mantox test is seen in a. pt taking immunosuppresents 158. postganglionic sympathetic supply to sweat gland is thru a. muscrinic receptors b. nicotinic receptors 159. pre-eclamptic pt,high bp,medicine given before anesthesia a. nitroprusside b. hydralazine ans: I marked b 160. which antiemetic is preffered in pts of chemotherapy a. odansterone b. metochlopramide ans: a 14

161. steady conc of doapmine is achieved in a. 9 min b. 2 min ans: a 162. drug interaction a. pharmacokinetic only b. pharmacodynamic only c. both pharmacokinetic and pharmacodynamic (answer) 163. ranitidine is different from cemitidine as a. it has less CNS toxicity (answer) b. less potent 164. systemic fungal infection is treated with a. amphoteracin b (answer),b. ketoconazole 165. alpha adrenergic causes a. mydriasis(answer) ,b. bronchodilation 166. 0.85% saline contains how much NACL a. 850mg/100ml b. 85g/L ans: a 167. dicumarol side effect a. platelets aggregation b. inc clotting c. dec clotting d. inc bleeding time(i think it was in the options) 168. eye infecton,diagnosed as HSV conjunctivitis...treatment a. trifluridine 169. dexamethasone as compare to hydrocortisone a. immunosuppression b. hyperglycemia can’t recall other options 170. vol of distribution is not affected by a. age ,b. gender (answer) 171. histamine releasing opioid a. morphine (answer),b. tramdol 172. morphine is used in -a. biliary colic -b. terminal cancer pain (answer) 173. thiazide diuretics acts on -a. DCT 174. cox 2 inhibiotor -a. meloxicam 175. antihypertensive acting directly on SA node a. verapamil(answer) b. nifidipine 176. mode of action of cephalosporin a. dec peptidoglycan synthesis b. inhibit transpeptidase (answer) 177. 2x2 table - a. chi square 178. study conducted to check temp change with general anesthesia over time(smthg like this)best way to calculate result a. student t test b. chi square c. ANOVA d. regression analysis 179. 2qstns regarding pts management and doctors behavious.... 180. on staining,cytokeratin indicates a. carcinoma b. to visualize metaplasia and dysplassia c. to visualize the contractile elements of cell d. to visualize the cytoskeletal structures of cell with impending death 181. rt shift of Hb dissociation curve ,if there is dec in a. PH

=-=-=-=-=-=-==-=-=-=-=-=-=-=-=-=-=-=-=-=-=-=-=-=-=-=-=-=-=-=-=-=-=-=-=-=- 15

( 2)Medicine & Allied 8th March 2016 MORNING by Dr Kunza Aslam Q no.1 : Pacinan corpuscle : A. high frequency vibration ( ans ) B. low frrequency vibration Q no 2 : What is used for traveller's diarrhea ? A. metochlopramide b. Diphenoxylate ( ans ) Q No3. plantar surface cutaneous innervation ? A. Sural nerve B Sephanous nerve C. posteior tibial nerve ( ans ) Q no 4. Esophagus : A. Passes theough right crus of diaphragm ( ans ) B. covered by peritoneum on anterior and right surface C. grooves posterior surface of liver Note: Peritoneum covers anterior and left side of abdominal part of esophagus. Ref: BD Churassia Q no 5. Aneurysm of aorta at diaphragmatic hiatus compresses A. azygous vein and thoracic duct ( ans ) Q no 6. HR = 70 , CO 5.6 L , EDV = 160 ml ,, calculate EF ? a. 0.5 L ( ans ) b. 0.05L Q nO 7. female met an accident , her son and husband died , afterwards she had night mares and amenorrhea . A. increased ACTH ( i marked ) B. low CRH C. low dopamine --> It is an Asim bcq but that's key was " increased CRH" Q NO. 8: female met an accidnt , whiplash injury ,, damaged C5 and C6. what will be affected a. flexion at elbow ( ans) past paper question Q no 9 : stab wound at the right of xiphisternum at the level of 6th costal cartilage , will penetrate which heart chamber or major venous channel ? a. right atrium B . Right ventricle C. IVC Q no 10: Fetal heart differs from adult heart by ? A. left atrium has more pressure than right atrium B. septum primum and secundum fuse to complete separation between common atria C. fused endocardial cushions divide right and left atrioventricular canals (I marked ) D. sinus venosus carries blood from to arch of aorta Q no 11. Antacid decreasing gastric motility a. Alluminium hydroxide ( ans ) Q no. 12: lithium is a toxic antipsychotic. when should it be stopped ? ( dont remember the exact statement ) a. when levels more than 1.2mmol B. coarse tremrs C. nausea and vomiting D. hypothyroidism 16

Q no 13: causing patchy demyelination and peripheral neuropathy ? A. Lead B. Co Q No 14. maximum renal tubular transport maximum (Tm) is for : A. sulphate B. lactate C. urate D. Phosphate Q N0 15.. female breast feeds her child . what would happen due to continuous suckling ? A. failure of primordial follicles to mature (ans) b. decreased prolactin Q no 16. in X ray right border ofheart is formed by A. SVC B. right Atrium Q no 17. bile salts are absorbed from A. ilium Q no 18. severe steatorrhea will result from A. terminal ilium resection B. total pancreatectomy (ans ) Q no 19. amino acids are absorbed from kidney via A. primary active transport B. secondary active transport (ans) Q no 20. Common btw diazepam and chlorpromazine A.sedation B.extrapyramidal side effects C. Parkinsnism D. Dependence E. Skeletal ms relaxation past paper q. Q no 21. anti emetic with highest bioavailability A. metoclopramide (ans) B. ondansetron Q no 22. vessel lying close to LAD , prone to injury during surgery is A. great cardiac vein (ans ) B. middle cardiac vein C. oblique cardiac vein Q no 23: purine metabolism end product : A. uric acid Q no 24 : Pt on ATT , cant differentiate btw red and green . which drug is responsible ? A. ethambutol ( ans ) B. INH Q no 25 . regarding pertussis most likely ? A. infants are susceptible from the time of birth B. DPT has no protective value c. Nasal and bronchial secretions are highly infectious ( I marked bcz maternal immunity is protective for first 6 months) Qno 26: sympathetic post ganglionic fibers release Ach : A. sweat glands (ans ) Q no 27: chemotaxis involves A. Migration (ans) B. Margination 17

Q no 28: in blacks , comlication of wound healing A. keloid formation (past paper q ) Q no 29. most aggressive CA : A. SCC B. BCC C. melanoma (ans ) Q no 30. Ventricles are completely depolarized in A. QRS B. ST segment (ans) past paper q Q no 31 : proteins dont appear in urine because of A. basement membrane (ans ) B. mesangium c. podocytes (nothing about negative charge mentioned ) Q no 32. tachyphylaxis A. rapid decrease in responsiveness to drug (ans ) B. radual decrease in response C. shown by morphine Q no 33. after cell injury , cellular swelling occurs due to A. water comes inside the cell from surroundings ( ans ) B.protein accumulation in cells C.lipofuscun accumulation D.glycogen break down Q no 34. RBC antigens A. secreted in saliva ( ans ) b. are agglutnins in hemolytic reactions nothing about glycolipid or glycosphingolipid mentioned EXPLANATION: antigens areAgglutinoGENS antibodies are AGGLUTININS Q no 35 . Asian man develops asian kind of cholangitis. he is at risk of developing cholangiocarcinoma . cause ? A, oncocerca volvulus B. chlonorchis sinensis ( ans ) Q no 36 . measels will cause A. Subacute Sclerosing pan enencephalitis ( ans ) B. progressive multifocal leukoenencephalopathy Q no 37. 22 yr old sexually active , reported to have been in contact with syphilis positive , how would his physiciann know about him having contacted the disease ? A. chancre on penis B. gumma on testicular biopsy C. condylomata accuminatum ---> answer is chancer.. although biospy would be diagnostic , but first go for non invasive procedure. Q no 38 . trachoma caused by A. chlamydia trachomatis Q no 39. Sulphur containing amino acid : A. methionine ( ans ) Q no 40. Chandkians scenario : Dr divided some group on age bases and then randomly allocated them in different groups 18

A. Stratified random sampling Q n 41. Temporal lobe lesion will affect most ? a. Memory ( ans ) B. olfaction Q no 42 : Pt has left hand tremors and increased propensity to fall towards left side . where is the lesion ? A. Left cerebellar hemisphere (ans ) B. Right cerebelar hemisphere Q no 43, primary brain vesicle A. Diencephalon B. Mesencephalon ( ans ) C. metencephalon D. prosencephalon E. myelencephalon Q no 44. X ray shows lobar pneumonia . How to diagnose ? A. sputum culture ( ans ) Rabia ali MCQ Q no 45 : what is allele ? A. non identical genes at same locus RAbia Ali Mcq Q no 46: hemorrahge . after compensation what will decrease ? A. heart rate B. cardiac contractility C. venous capacitance ( I marked ) D . BP Q no 47 : increase in EF decreases ? A. ESV Rabia ali mcq Q no 48. what causes conversion of chondrocytes to osteocytes.. ( dont remeber the exact stem , but it was like what increases the bone growth ) A, somatotrophs ( I marked ) B. thyroxin Q no 49 : BMR in resting condition maintained mainly by ? A. liver ( I marked ) B. Heart C. Kidney D.Brain Q no 50 : most common cause of pulmonary and DEATH ? A. thromboembolism ( I marked as per previous discussions on group ) B fat embolism Q no 51 : Pt post MI develops cyanosis. what is the cause ? A. ventilation perfusion mis match ( the best option among other options which pointed towards pulmonary embolism ) Q no 52. Intervertebral discs A. make 1/ 4th of the lengthof vertebral column ( i marked ) B. made of hyaline cartilage c. something about annulus fibrosis composition D. something about size of discs during day and night Q no 53. decreased GABA in Globus pallidus and substantia nigra ? A. Chorea B. Athetosis( ans ) 19

C. parkinsonism Q no 54. Immunological test done for A. hydatid cyst ( ans ) B. amoebic liver abscess Q no 55 : Child with Down sndrome. One of the parents has chances of having ? A. Robertsonian translocation ( i marked ) B. reciprocal translocation C. mosaicism Q no 56 . Central chemoreceptors respond to A. CSF ph B. pCO2 in cerebral veins ( ans ) Q no 57. patellar tendon tapped. what will appen ? A. increased 1b afferent firing B. quadriceps femoris will contract ( ans ) Q no 58 : Alpha agonistic activity will cause ? A. contraction of radial muscles of iris ( ans ) Q no 59 : Heparin primarily works through : A. anti thrombin 3 ( ans ) B. factor X past paper q Q no 60. most common site of obstruction for hydrocephalus A. Aqueduc of sylvius ( ans ) B, inter ventricular foramen C. foramen of Luschka Q no 61 : Ascending reticular formation located in a . floor of aqueduct of sylvius ( ans ) b. Cortex C. Hypothalamus past paper q Q no 62. Middle meningeal artery A. passes through foramen spinosum (ans ) B. divides at pterion C. causes subdural hemorrhage Q no 63. Human blood : A. platelet more in no than RBCs B.RBC larger than WBC C. RBCs are biconvex D, Iron is mainly in haemoglobin (ans) Q no 64 : In parenchymal organs , iron is stored as A, Ferritin ( ans ) B. hemosidrin Q no 65 . Pt has facial congesion , lung CA diagnosed to be small cell (oat cell) CA , it will show increased A. ACTH ( ans ) B. PTHrP Q no 66 : Patient taking propranolol develops increased PR interval . what is the cause ? A. Drug induced 1st degree heart block ( ans ) Q no 67 : Pt given one litre N/S , what will happen? A . decreased urinary osmolarity B.increased plasma osmolarity C. increased urinary osmolarity 20

I dont remember the rest of options. it is quite a possibilty that the right one is missing here. osmolarity remains same , aisa koi option nahi tha. Aongst all , A looked best to me and i marked that. Q no68. Free nerve endings ? A. non encapsulated Q no 69. Osmolarity of Human plasma ? A. Similar to 0.9 % N/S ( ans ) B. similar to 0.5 % Dextrose Q no 70. Structures which prevent communication of un wanted materials between cells ? A. Tight junctions ( ans ) B. zona adherens C. desmosomes D. gap junctions Q no 71. Common btw skeletal and smooth muscles ? A. increased Intra cellular Ca before contraction (ans ) B. both are striated C. sarcomeres Q no 72. long bone mostly fractured ? A. tibia ( ans . bcz it is asuperficial bone , most part uncovered by muscles ) B. femur Q no 73. post ganglionis sympathetic fibers are present in A. cervical B. thoracic C. lumbar D. sacral E. all spinal nerves ( ans ) it is a Rabia Ali q. Q n 74. First bone to ossify in inrauterine life ? A. femur ( I marked ) B. Clavicle Q no 75. Smoking is associated with A. thromboangiitis obliterans ( ans , it is also called BUERGER disease ) B. takayasu C.giant cell arteriti D.wegners granulomatosis E.PAN Q no 76. which of these is not synthesized by post ganglionic sympathetic fibers ? a. ach B. dopamine C. nor epinephrine D. Epinephrine E. L.Dopa ( answer ) Q no 77 : 90 % of anthrax infections are associated with A. Cutaneous lesions ( ans ) past paper q. Q no 78 : Bromocriptine reversed amenorrhoea . how ? A. acts as dopamin agonist ( ans ) B. increases prolactin Q no 79 : first factor that goes in Vit K deficiency ? A. factror 7 ( ans ) Q no 80 : Leydig cells 21

A. make sperm testis barrier B. mainly reside in seminiferous tubules C. have high level of SER ( i marked ) D. cause sperm maturation --> Sertoli cells make blood testes barrier and provide nourishment to the sperms Q no 81 : structure that passes through posterior mediastinum ? A. thoracic aorta ( ans ) B. SVC C. thymus Q no 82 : thalasemia suspected in fetus , how to diagnose ? A. Chorionic villous sampling ( I marked ) B. maternal blood for fetal RBCs or antigen ( dont remember exactly ) C. materal urine for fetal hb D. maternal Hb electrophoresis no 83: Primigravida taking iron tablets , lots of fresh vegetables and juices .. what deficiency she is at risk of developing ? A. Ca ( i marked ) B. vit B12 C. folic acid D. vitamin E E. vitamin B1 Q no 84 : Pt awake with eyes closed, EEG on occipital leads will show A. alpha waves ( ans ) Q no 85 : Fisherman only eating fish and rice , anemia. MCV 106. Some neuropathy sign.stool shows ova. Cause ? A. Diphylobothrium latum ( ans ) Q no 86: Scalenus anterior A. Phrenic nerve passes anterior to it.( i marked ) B subclavian vein passes behind it C.attached to outer border of 1st rib Q no 87 : Main support of uterus. A .transverse cervical ligamant ( ans ) B.round ligament C.broad ligament Q no 88: antioxidant vitamin A. vit e ( ans ) B. Vit A C: Vt C q no 89 : not a part of natural immunity A . plasminogen ( ans ) b. interferon Q no 90 ; .person standing from sitting position , Heart rate increases due to A.dec firing rate frm baroreceptor ( ans , confirm ) B. increased firing rate from baroreceptors Q no 91 : Viruses cause cancer by doing alteration in A . Nucleic acid ( I marked as it is another name for DNA and viruses cause cancer by altering DNA and thus they alter protein synthesis ) B. oncogene C. proto oncogene Q no 92 : Pulmonary artery supplies Primary bronchus 22

secondary bronchus terminal bronchiole Alveoli ( ans ) Q no 93 : septic shock caused by A.gram negative bacteria ( ans ) Q no 94 : .hamartoma? A.totally benign ( ans ) B. presence of gestric tissue in meckel's Q no 95 : .myeloperoxidase present in A.neutrophills ( ans ) Q no 96 : Thorn prick on leg. Abscess develops A. Staph aureus ( ans ) Q no 97 : metastasis occurs due to .1loss of E cadherin ( ans ) 2. Migration of tumor cells Q no 98 : Medical ethics A. Moral of professional conduct ( ans ) B.same as hippocratic oath chandkians mcq , i guess Q no 99 : Pt with bee sting.after 4 hours , he has periorbital edema , Hypotension bp 100 by 6o. Pulse 110(delayed anaphylactic reaction ban rha tha ) what to give ? A. IM adreanline B. Hydrocortisone Q no 100 : Pulse pressure decreases in A. Increased capacitance of B. Dec stroke volume ( i marked ) b was best amongst all.. i dont remember the rest of options Q no 101 : Extra systole what will happen A."Dec pulse pressure bcz of decreased stroke volume" i cant recall the rest of optios. Q no 102 : . Trigeminal nerve A . Supplies temporalis ( ans ) B. Supplies angle of jaw C. Has 3 roots Q no 103 : blood supply of interventricular septum .a)anterior interventricular artery ( ans , as /3rd part is supplied by it ) b)post interventricular artery Q no 104 : Acute mild rejection of heart transplant. What will be the finding there ? A. Inc neutrophils B. Inc lymphocytes cant remember the other options Q no 105 :PPD test . Induration will have cells A. T cells and macrophages ( ans ) Q no 106 : Increased risk of cancer in HIV positive pt A. Uterine cervix ( ans ) Q no 107 : Pt pap smear hpv positive. After 2 years , increased nucleus size , prominent nucleoli . What is it A. Dysplasia Rabia ali mcq Q no 108 : After chemo, cancer cells die , nucleus and cytoplasm shows fragmentation. What is the mechanism? 23

Apoptosis ( ans ) Q no 109 : .Pt cant see the left temporal and right nasal fields. Where is the lesion ? A. Right optic tract. ( ans ) B. Left optic tract. C. Rt optic nerve Q no 110 : related CA in HIV A. Kaposi sarcoma Q no 111 : . Factor 8 is synthesized by A. Endothelial cells (ans) B. Hepatocytes Rabia ali mcq Q no 112 : . Echo virus causes A. Meningoencephalitis ( ans ) Past paper q Q no 113 : Female swelling infront of neck.. hypothyroid. Dr orders some immunological test. What is the diagnosis ? A. Hashimoto thyroidits ( ans ) B. lymphoma Q no 114 : Poliomyelitis A. Affects anterior horn cells (ans ) B.reaches cns through nerves Q no 115 : .Spinal cord B. In neonates ends at L3 ( i marked ) C. Central canal extends into the filum terminale D. Filum terminale ends at S2 Q no 116 : .Myxoid degenration A. Mitral valve prolapse ( ans) B. Libman sac C. Marantic D.infective endocarditis Q no 119 : Correct sequence of events ? A. Damaged valve, thrombus , perforation , emboli B. Damaged valve , perforation , thrombus , bacteremia C. Damaged valve , bacteremia , thrombus , perforation D. Bacteremia , thrombus , perforation , emboli ( bht zyada resemble krti thi options ) Q no 120 : pt with cancer T4N1M1 , such that these pts have low survival rate. What will be common to such pts ? A. Cachexia ( i marked ) B. HCC C. cervical CA Q no 121 : Scenario of TB. Like X ray p kuch findings thi .. easy scenario tha. They asked about type of hypersensitivity ? Type 4 hypersensitivity (ans ) Q no 122 : what is the landmark for pudendal nerve block ? 1.sacral promontory .2.ischial spine ( ans ) 3.ischial tuberosity 4. Pubic tubercle Q no 123 : Winging of scapula caused by damage to which nerve ? Long thoracic nerve ( ans ) 24

Q no 124 : Pt with portal HTN , ascites and dilated abdominal veins. What vein is involved ? A . IVC B. Portal vein ( I marked ) Q no 125 : Pt given oral anti coagulant. How to monitor A . PT ( ans ) B. aptt INR was not in options Q no 126 : Child witj recurrent knee joint swelling and pain. Faxtor 8 was 57 % , i guess and factor 9 was 8 % ya 18% (dont remember exacctly). What is the disease ? A. Christmas disease ( ans) B. Hemophilia A C. vWb Q no 127 : 20yrs Female with anemia , hb 6 , retics 10% , spleenomegaly ,peripheral smear showed microspherocytes. What test to do. A. RBC survival studies ( i marked ). B. Uric acid test C. Coombs test D. G6PD assay ( Scenario is of Hereditary spherocytosis ) Q no 128 : intravascular hemolysis : A. Dec heptoglobin levels ( ans ) Q no 129 : Ph 7.5 , hCO3 30 , pco2 40mmHg ? A. Uncompensated metabolic alkalosis.( ans ) B. Partially compensated metabolic alkalosis Q no 130 : Tumor due to gene amplification ? Neuroblastoma ( ans due to amplification of N-myc gene ) Retinoblastoma past paper q Q no 134 : . Brain venous draiange A. Dural sinuses( ans) B. Diploic veinx C. Emmissary veins D . Internal jugular vein E. Petrosal venous sinus Q no 135 : At which stage nuclei disappear ? A. Proeryhtroblast B. Erythroblasts C. reticulocytes ( i marked ) i dont remember the other options Q no 145 : Carbon laden macrophages A. Pneumoconiosis B. Anthracosis ( i marked ) Q no 146 : Female with some resp issue , non smoker. X ray shows increased translucency. Alpha 1 antitrypsin deficient. What will be most affected ? A. Alveolar duct ( ans ) B. Terminal bronchiole Baqi sare alveolar duct se oper wali respiratory divisons thi or koi alveoli ki option nahi thi ( alpha anti trypain defficiency causes panacinar COPD ) Q no 147 : obstructive pattern disease diagnosed by A. FEV1/FVC less than 75 (ans ) 25

Q no 150 : Obstructive lung disease ka scenario tha. X ray showed increased translucency . what will increase ? 1. FRC ( i marked) 2. Vital capacity 3.FEV1 /FvC Q no 151: Pt with septicemia, on ventilatory support. More and more ventilatory pressure required to maintain ventilation.. x ray shows opacities. A. Diffuse alveolar damage ( i marked ) B. Intra alveolar neutrophilic exudate c. Patchy atelactasis Baqi yad nahi Q no 152 : pt with lung fibrosis has dyspnea due to A. Dec diffusing capacity of O2. ( i marked ) B. increased airway resistance Q no 153 : What is the mediator of allergy ? A. 5 hydroxy tryptamine B. Bradykinin C. Adrenaline Histamin was not in options Q no 154 : .Melatonin A. Increased in day time B. Produced from tyrosine C. Increases GNRH release or something like that I cant recall other options. some say it has some role in puberty. it is better to read melatonin from somewhere Q no 155 : .diffusion capacity of O2 A. Less than N2 B. More than CO2 C. Decreases in exercise D. Decreases in lung fibrosis or something like that this question options are not properly recalled .. many people chose A but no one was sure about it. Q no 156 : Dr pt relationship. What is most important A. Mutual trust B.respect C.clarity D. Autonomy E. Clarity Q no 157 : Pt with HTN , LVH , Creatinine 0.8 , Urea ki b value di hoe thi , BSF 88mg% with urinary sugar ++ , Na 140 , K 2.5. Cause of HTN A. Hyperaldosteronism ( answer ) B. Essential HTN C.diabetic nephropathy D. Glomerulonephritis Rabia ali bcq Q no 158 : Cotton wool spots A. Diabetic retinopathy (ans) B. Hypertensive retinopathy Q no 159 : Tympanic reflex A. Frequency 20 decibles something like that B. Requires continuous chain of ossicles 26

C. At very loud voices I dont remember the other options Q no 160: 250Hz frequency detected by A. Pacinian ( answer) Baqi options me baqi receptors the Q no 161 : .4 yrs child with meningitis plus rash ( meningococcal meningitis ka scenario ban rha tha ) treatment A. 3rd generation cephalosporines B. 2nd generation cephalosprns. C. Benzylpenicillin 4. Eryhtromycin 5. 1st generation cephalosporins Q no 162 : Kid with meningitis then develops bilateral deafness A. Damage to the neural pathways B. Rupture of tympanic membrane C. Damage to organ of corti D. Altered level of Na in scala media ,something like that Q no 163 : CSF from normal human being A. More potassium than plasma B. Less glucose than plasma C. Ph comparable with plasma D. Something about osmolarity Q no 164 CSF? A. Proteins from 20 to 40mg per dl. B. Glucose 120mg /dl Baqi options nahi yad. I marked A Q no 165 : Exudate contains proteins A. 2 to 2.5mg% B. 2.5 to 4mg% C. More than 4mg% Q no 166 : Dialyzing fluid has more than plasma A. Glucose Thori different wording thi. Q no 167 : . Child with loose skin (shaed, exact lafz nahi yad) with easy bruisability and defective procollagen 3.what can be the most striking symptom / complication ? A . Hyperextensible skin B. Hypermobile joints C. Problm in articular cartilage. D. Rupture of large vessels Q no 168 : Most common cayse of HCC in developing world A. Hep B plus hep C B. Hep B. C. Hep C Q no 169 : Vasoconstriction caused by A. ADH ( answer) B.aldosteron C. Renin Angiotensin was not in options Q no 170 : BP increased by A. Alpha 1 agonist 27

B. Saralasin C. Phenoxybenzamine Q no 172 : Active transport basically involves A . Downhill transport B. Uphill transport C. Atp breakdown something D. Carrier proteins E. Carrier protein saturation Q no 173 : .at site of bifuraction of common carotid , internal carotid artery related to external carotid A.posterior B. Anteromedial C. Lateral Q no 174 : External carotid differs from internal carotid A. Gives branches in neck Q no 175 : Trauma to face , facial asymmetry , cant close eyes. Nerve damaged A. Facial ( ans) Q no 176 : Protrude jaw A. Lateral pterygoid ( ans) Q no 177 : Lymphoid mass with stratified sq epi A. Palatine tonsil ( ans ) Q no 178 : Howship lacunae contain A. Osteoclasts ( ans) Q no 179 : intestinal basic electric rhythem A. Alters in respone to the hormones in circulation ( ye exact statmnt nahi he) B. Constant in duodenum and and alters in other segments C. Diminshes during mechanical digestive movemnts D. Or b ajeeb o ghareeb options thi. Better to read the topic Q no 180 : Duodenum A. Decreases gestruc emptying in response to fatty meal B. Secretes bile in response to fatty meal Q no 181 : Dudenal secretions in respomse to A. When ph less than 4.5 B. High ph Q no 182 :Not HLA associated A. Mysthenia B.graves C. Type 1 DM D. RA E. SS Q no 183 : Loud S1 A. Mitral insufficuency B.aortic stenosis C.RBBB D. PR interval upper normal limit ,( something like that) E. PR interval lower normal limit ( somethng liek that) Q no 184 : Passes infront of pulmonary artery A. Phrenic nerve ( I marked ) B. Bronchus C. Recuurent laryngeal nerve 28

D. Pulmonary veins Q no 185 : Vessel damaged due to close proximity to phrenic nerve A. Pericardiophrwnic artery Q no 186 : Fragile X syndrome relatd to A . Trinucleotide repeats (ans) B. X chrmosome Q no 188 : more selective for COX 2 than for COX 1 ? A. Meloxicam ( ans ) B. piroxicam celecoxib was not in options Q no 189 : Ischemia from hepatic flexure to middle of transverse colon A. Middle colic ( I marked ) B.ileocolic C. Sup mesenteric artery D. Marginal artery Q no 190: Low Ca , high Phosphate, normal alkaline phosphatase A. Renal failure ( answer ) Q no 191 : Boy with fracture , low Ca (6 tha shaid ). Develops carpopedal spasm on taking BP... Ca being excreted in urine ( Some value of Ca in urine was given ) A. Vit D deficiency B. Hypoparathyroidism ( i marked) Q no 192 : Superior Parathyroid gland A. Supplied by sup thyroid artery B. Lies at middle of posterior border/ surface of thyroid lobe at level of 1st tracheal ring (ans ) C . More variable in postion D. Just anetrior to pretracheal fascia Q no 193 : Pus contains A. dead bacteria B. dead neutrophils ( I marked ) C. dead lymphocytes Q no 194 : peripheral nerve damaged , nerve will regenerate at the rate of A. 0.1 mm/day B. 0.3 mm/ day C. 10mm/day Q no 195 : Glucocorticoids cause A. decrease peripheral utilization of glucose ( ans ) Q no 196 : Pannus is a feature of A. RA (ans ) Q no 197 : Heart is most susceptible to fibrillation ? A. during refractory period B. peak of action potential C. starting of action potential D. Near the end of action potential ( I marked ) Q no 198 : Pt has pericardial friction rub with deranged RFTs . what is the cause of pericarditis ? A. serous ,B. Fibrinous ,C. exudative Q no 199. Pt with damaged valves develops fever , splinter hemorrhages ( scenario of IE ), what is the cause ? A. strept viridanse (ans ) B. Staph Q no 200: tobacco smoking affects organogenesis during ,A.1 to 3 weeks ,B.3 to 8 weeks(ans) C.after 12 weeks =-=-=-=-=-=-=-=-=-=-=-=-=-=-=-=-=-=-=-=-=-=-=-=-=-=-=-=-=-=-=-=-=-=-=-=-=-=-=-=-==-=-=-=-= 29

(3) Medicine-8th March Evening By Dr.M Tariq Khan & Mudassir Bangash(200) 1. Which structure is embeded in temporal lobe----->>>insula 2.Pseudomembranous colitis is caused by---->>> clostridium difficle 3-Alpha-2-agonist used in ICU for sedation a. Clonidine b. Phentolamine c. Phenoxybenzamine d. Prazosin e. Dexmedetomidine (Ans) 4-Ureteric bud is formed as a part of a. Mesonephric duct (Ans) b. Paramesonephric duct c. Pronephros d. Cloaca e. Metanephros 5-Coccygeal ligament start at---->>> L1 6-About Cornybacterium Diphtheriae select correct One a. Spore forming b. Produce Exotoxin (Ans) c. Penicilline Resistance 7-A 40 years Old man (Hafiz Ullah) from Sibi Balochistan with fever and Lymphadenopathy with the H/O of sleeping on floor and often bite by flies. On Peripheral bood film Unicellular structure can be seen. Dx a. Kalazar (Ans) b. Malaria c. Infectious Mononucleosis 8-Clavulinic Acid is use with Penicilline to a. Decrease penicilline Resistance b. It inhibit beta lactamase (Ans) 9-Cimitidine ------>>> Inhibit Hepatic Metabolism 10-Drug that increase LES tone and Increase Gastric emptying------>>> Metoclopramide 11-Ondasentron M/A was asked a. Block dopaminergic in ChemoTriger Zone b. 5-HT3 antagonist (Ans) 12- A scenarion of Man which can't differentiate between colours.Which structure will be absent a. Rods b. Cones (Ans) c. Retina d. Optic Tract 13-Vitamin A deficiency first sign----->>> Night Blindness cpsp demo Qx 14-Thiamine cause ------>>> Peripheral Neuropathy (Rabia Ali Qx) 15-TGs highest in ------>>> Chylomicrones 16- A dental surgeon is having pain in his back and MRI shows involvement of Sciatic nerve. Which intervertebral diac will be involve------>>> L4L5 17- Irreversible step in Glycolysis------>>> Glucose to Fructose-6-Phosphate 18- Regarding S3 heart sound------>>> Rapid ventricular filling 19-Regarding S4 heart sound------>>> Atrial Systole 20- A Qx of Middle Meningeal Artery about its supply..Option was confusing can't reacall 21- A scenario of Turner syndrome. Genotype was asked----->>>XO 22-A case of Kilnefilter Karyotype------>>> XXY 23-Partial mole genotype was asked----->>> 69XXY 24-Extra-Ocular mucles of eyes are controlled by which Loop a. Association loop b. Orbital loop c. Occulomotor loop (Ans) 25- Nerve asked, loss of sensation in lower teeth, chin and lower lip a. Inferior Alveolar nerve (I did) b. Madibular nerve 26-Retromandibular vein is formed by junction of Maxillary vein and (#PPQ April 2014) a. Facial vein 30

b. Lingual vein c. Maxillary vein d. Superficial temporal vein (I did) e. Transverse Facial 27-Intracranial and extracranial veins are joined by a. (Ans) b. Cavernous sinus c. Dipolic veins 28-Systamic Lupus Erythematous typical Scenario , Investigation of asked (ANA was not in option)------>>> Anti-dsDNA 29-A women was watching a cricket match and develop malar rash. A scenario of SLE and complication was asked------>>> a. Pericarditis (I did)/ b. Pulmonary fibrosis 30-A 50 years Old man with arthritis from last 5 years. Left and right cardiomyopathy and raised Ferritin. Which other investigation will help in Dx a. Anti-centromere antibodies (I did) confirm it b. ESR more than 75 31-A 30 years Old multigravida suffer miscarriage at GA 12 week. Whcih will be best Regime a. Uterine evacuation (I did) Confirm it b. Uterine evacuation +FFP+Packed RBCs c. FFP+Evacuation 32-A man has to undergo teeth extraction and also using Warfarin, which test to be done a. PT,aPPT,INR (Ans , all three in one option was given) <> b. BT c. aPPT d. CT 33-Bleeding time and aPPT prolonged and PT normal------>>> vWD 34-Another Scenarion with Only BT was prolonged------>>> vWD 35-How Aspirin protect in Coronary disease------>>> Inhibit cyclooxigenase 36-Situation of SA node------>>>Upper portion of Sulcus terminalis 37-Thromocytopenia in Children due to Acute Viral infection is becausw of a. Pooling of platelets in spleen b. Antibodies against platelets (Ans) c. Immune Complex in blood 38- A child with Parasite infection, Which cell will increases------>>> Eosinophils 39-APC in Intestinal mucosa------>>> M Cells 40-Secretions with high PH (Ganong BCQ) a. Gastric juice b. Pancreatic juice (Ans) c. Gall bladder secretions d. Salive 41-Esophagous epithelium was asked (cpsp demo Qx)------>>> Upper 1/3 sq. 42-Esophagous narrowest at------>>> Cricopharyngus 43-Enocervix with squamous epithelium------>>>Metaplasia 44-Benign tumor -a. Warthin tumor 45- Characteristic feature of Malignancy -a. Invasion (Metastasis was not in option) 46-Another Qx of Malignancy I think histopathological feature was asked a. Pleomarphism (Ans) b. Inc N:C ratio 31

Metastasis and invasion was not mention in that Qx 47-Alleles are -a. Non-identical gene on same locus 48-True about Nucleus a. Eucromatin is transcribtively active and form RNA [I did] b. Hemotoxylin is blue dye which stain nucleus 49-Most radiosensitive tumor a. Bone b. Cartilage c. Lymph node (I did) 50-In long bones from secondary centre of ossification what is formed a. Epiphyseal plate (I did) confirm it b. Epiphysis c. Metaphysis 51-Insulin is blocked by -a. Beta blockers 52-A young boy with undescended testis, it could lead to a. Malignancy (Ans) b. Impotence 53-50 years old female with normal living. What will happen post-menopausally------>>> Increase FSH & LH 54-Cerebral blood flow at rest a. 25ml/100mg/min b. 50ml/100mg/min (Ans) c. 75ml/100mg/min d. 100ml/100mg/min e. 150m/100mg/min 55-Upper and lower limits of cerebral blood flow autoregulation a. 25-150ml/min b. 50-150ml/min (Ans) c. 50-200ml/min d. 75-200ml/min 56-In some children Dorsalis pedis Artery is missing congenitally. Then dorsum of foot will be supplied by which artery a. Plantar Arch b. Lateral plantar artery c. Peroneal artery (Ans) d. Medial plantar artery e. 1st metatarsal Artery 57-Itching will stimulate (Ganong BCQ) a. Dorsal column b. touch receptors c. C fibers (Ans) d. A alpha e. B type fibers 58-Sensation in periphery a. All must have to pass through reticular formation (I did) confirm it b. Half will pass through reticular formation c. Never pass through reticular formation d. Depending on modalities some pass while other Bypass (???) 59-Masseteric fascia derived from a. Carotid sheath b. Pretracheal fascia 32

c. Prevertebral fascia d. superficial layer of Deep cervical fascia (Ans) e. Buccopharyngeal fascia 60-A Child got blow at elbow and elbow joint displaced. And loss of sensation over little finger. Structure involved a. Medial collateral ligament b. Ulnar collateral ligament (Ans) c. Medial meniscus 61- A 35 years old man die suddenly. Autopsy finding nill. Toxicology shows cocaine and its metabolite in blood. Cause of death a. Myofibrillar disarray b. Contarctile band (Ans) c. Myocarditis 62-Ganglion found at T7 vertebra a. Pteregopalatine b. Geniculate c. Thoracocervical (Ans) 63-Edrophonium most common adverse effect a. Diaphoresis b. Dry mouth c. Nausea (Ans) d. Euphorea e. Sweating 64-Damage to first lumbar vertebra will lead to a. Conus Medullaris Syndrome (Ans) b. Damage to spinal Cord c. Damage to sympathetic system 65-In feamle neck of bladder lie a. Lie above urogenital diaphragm (Rabia Ali BCQ) 66-Most significant cause of atheroma a. Alcohol b. Diastolic hypertension c. Type 2 personality d. Choleatatoma (I did) 67- The life style patteen of healthy persons from 20 to 30 years of age are studied. A subset of these persons have a lifestyle characterized by consumption of a lot of pizza and very little physical activity. Which of the following tissue changes is most likely to develop in these subset of persons as a consequence of this lifestyle. (Qx is taken from USMLE forum where Key is D) a. Fatty Metamorphosis of liver b. Pancreatic fat Necrosis c. Fatty degeneration of myocardium d. Hypertrophy of Steatocytes (Ans) 68-Mediators of pain in acute inflamation a. Bradykinin and PGs (Ans) b. IL1 and TNF alpha c. IL6 69-A lady presented with pain in shoulder and can't ADDuct and Medially rotate a. Teres Major (I did) b. Teres minor c. Supraspinatus 33

d. Infraspinatus e. Subscapularis 70-A child with infection blood is need for lab studies. Vensection done. Which structure will be damage a. Saphenous vein (Ans) b. Common peroneal nerve c. Superficial peroneal nerve 71-A man with Bp of 200/100mmHg and glucose level of 200mg/dl, which drug you will give a. Linisopril (I did) b. Furosamide c. Amlodipine d. Propanolol 72-Digoxin is DOC for a. Atrial fibrillation 73-A scenario with P wave was replaced with some irregular wave and irregular R-R and irregular pulse. Radial rate is less than apex beat a. Atrial fibrillation (Ans) b. AV nodal block c. Ventricular arrythmias d. Ventricular Fibrillation e. Atrial fluuter 74-A lady with Hb 6, MCH 36, MHCH 29, MCV 79 fl, HbA 94% and HBA2 4% a. Iron deficiency anemia (Ans) b. Thalasemia trait c. Alpha thalasemia 75-Point mutation cause a. Sickle cell Anemia (Ans) b. Alpha thalasemia c. Huntington disease d. beta thalasemia 76-In EKG Heart rate is 120/min a. R-R interval will be 0.5 sec (Ans) b. QRS complex will be less than 0.02 sec c. T wave will be greater than 2 mv 77-A lady with mild fatigue since 5 years Hb 11, MCV 104 fl, AST 80 and ALT 85 all other values are normal. Heptoglobulin 88 Reticulcytes 0.7& Billirubin1.2 a. Hepatitis C infection (I did) b. Vit B12 deficiency 78- A man with itching and jaundice Direct billirubin was raised a. Typical case of extrahepatic cholelithiasis 79- Mechanoreceptors of Warm----->>>Long receptive field 80-Cutaneous and Gracillis damage will result---->>>Asterognosia 81-A lady with 6 week of German measles---->>>Cataract 82-Mother gives birth to a child with cataract. Which infection she had during pregnancy------>>>Rubella 83-WBC adhesion molecule------>>>FLA-1 84- A Qx of transplant (nasty one) a. if both donor and recipient with CMV antibodies positive should not be done transplantation b. Cyclosporin should be given after transplant to suppress CD8 cell proliferation 34

c. HLA-1 is more important than HLA-2 (Ans Ref Goljan page 41) 85-Norepinephrine is released at a. Presynaptic nerve endings (Ans) b. Post ganglionic sympathetic c. Parasympathetic Preganglionic d. Preganglionic Sympathetic e. Presynaptic adrenal medulla 86-During wound healing strength is provide by (exact statment) ---->>>Type 3 collagen ( Ref Kaplan Anatomy) 87-Articular surface has which cartilage----->>>Hyaline 88-Elastic cartilage is found at------>>>Epiglottis 89-Preganglionic Presympathetic fibers----->>> a. B type (I did) b. A type c. C type 90-Reticular connective tissue is present in a. Tonsil (I did) b. Dermis c. Salivary 91- Regarding Vitamin A, Retinoic acid act as a. Vitamin A (I did) b. Is toxic 92-Reagarding QRS complex----->>>Occur prior to ventricular systole 93-Paraneoplastic syndrome---->>>Small cell lung Ca (I did)/ Pancoast tumor 94-Diastolic murmur and weak radial pulse---->>>Aortic Regurgitation 95-A man with chest pain from last 3-4 hours which enzyme will be raised a. Cpk-MB (Ans) b. CPK c. LDH 96-Glucocorticoides does a. Deamination of Amino Acids b. Dec uptake of Amino acid in liver c. Dec glucose utilization (Ans) d. Inc fatty Acid in blood e. Inc protein Anabolism 97-A man having Hep C on histology eosinophilic granules in cytoplasm with compact chromatin a. Apoptosis (Ans) b. Necrosis c. Fibrosis 98-Exophthalamus is due to a. Inc TSI (Ans) b. Inc T3T4 c. Inc thyroglobulin 99-Which microorganism cause Malignancy in GIT----->>>H.pylori 100-Apoptosis is by------>>>Activation of caspases 101-In Thyrotoxicosis (inc thyroxine) what metabolic activity happen a. Gluconeogenesis (I did) b. Lipogenesis c. Protein synthesis d. Atherosclerosis 35

102-Which drug is inducer------>>>Phenobarbitone 103-One Qx was about inhibitors----->>>Ketoconazole 104-A 2 years boy presented with pallor and bowel disturbance. No physical findings seen. What lab investigation to be done a. Serum Vit12??? b. Intrinsic factor (I did) confirm it 105- Benzodiazepine cause----->>>Dependence 106-Thiazide Diuretics cause------>>>Hypokalemia 107-Osmotic drugs act on------>>>PCT 108-Maximum glucose and Amino acid are reabsorbed in ----->>>PCT 109-Resection of ileum more than 100cm will cause----->>>Dec absorption of bile salts 110-Supply of midgut, Root value was asked a. T8-T12 (I did) b. S2S3S4 c. T6-T9 d. Greater spanchnic nerve 111-A patient with tremor, gait disturbance, bradykinesia, rigidity. Nueron affected------>>>Substancia Nigra 112- PICA damage, blood supply of which part will be affected a. dorslateral of medulla [Ans] b. Pons c. Midbrain d. Cerebellum 113-Aphasia due to which lobe involvment [<>] a. Temporal lobe (Ans) b. Parietal lobe c. Occipital lobe 114- In summer in a fasting man concentrated urine is formed due to a. Secretion of ADH (Ans) b. Sweating c. Dehydration 115- In severe dehydration a. Whole body fluid will be low (I did) b. ECF will be low c. ICF will be low 116-A man with heavy sweating along with that he took 2L of water. What will increase a. IC volume (I did) b. EC volume c. IC osmolarity d. EC osmolarity 117-Plasma osmolarity is decreased by a. Vasopressin (I did) b. Isotonic saline c. 20% Albumin d. Hypertonic solution 118-A boy with protein greater than 3.6g with proteinuria and edema---->>>Dec colloid osmotic pressure 119-About neutrophils------>>> a. Active phagocytes in blood (I did) b. Move in and out of circulation 120-ESR decrease with ------>>>Inc Albumin (Ans) 36

121-One Qx of Autoimmune hemolytic Anemia very confusing and loooooooong one can't recall it 122-O2 releasing from Oxyhemoglobin is decreased by----->>>Dec temperature 123-Clinical cause of cyanosis------>>> a. Dec oxygen in blood b. Dec Hb saturation c. Inc conc of deOxyhemoglobin (I did) confirm it 124-A person smoker and working in construction , bilateral diaphragmatic lung lesions on CXR------>>>Abestosis 125-In bronchoscopy which part of bronchus will be seen first a. Right lower bronchus (Ans) b. Upper right lobe c. Apical bronchus 126-A person lying on his neck, aspire a foreign body. Will lodge in ----->>>Apical portion of right lower lobe 127- A man working in tyre and repairing. Presented with carcinoma. Cause don't remember exact scenario) a. Alanine dye b. Smoking c. Aromatic amines ( Ans???) d. Nitrates 128-Virus cause damage by------>>>>Altering protein synthesis 129-2*2 or four values are represented by----->>>Chi-square chart 130- Study of disease against non-disease and looking for risk factors---->>>Case-Control study 131-Four basic pillar of medical ethics---->>>Autonomy, Justice, Beneficence, non-maleficence 132- Inc blood flow to muscle----->>>Local metabolite 133-Regarding sigmoid colon 《《《Option A wrong coz continue as rectum at S3》》》 a. continue as rectum at S5 b. Is infront of pelvic brim /Begins at pelvic brim (Correct Answer) c. It does not have tenia coli d. Recieve inervation from S1-3 e. In sigmoid no mesocolon 134-Left renal vein is infront of----->>>Aorta (Rabia Ali BCQ) 135-NK cells------>>>Kill virus with prior sensitization 136-Structure loop around arch of aorta------>>>Left Recurrent Laryngeal 137- Left ventricular failure is caused by a. Aortic valve disease (Ans) b. Mitral stenosis c. Cor pulmonale d. Pulmonary valve disease 138-Which hormone act via cAMP Mechanism a. Glucagon (Ans) b. Oxytocin c. Insulin d. Estrogen 139-In Auscultation 3rd intercostal space left to sternum a. Aortic A2 (I did) b. Aortic A1 c. Pulmonary valve sound 37

d. Mitral valve (Aortic A2=Erb point=3rd ICS Left) 140-A 25 years old boy complaining of dyspnea on lying down--->>>Retrosternal goiter. 141- A patient with COPD and Po2 55, HCO3 26, pCO2 49 and PH 7.3 a. Type I Respiratory failure with uncompansated Respiratory Acidosis b. Type II Respiratory failure with uncompansated Respiratory Acidosis (I did) c. Respiratory Acidosis d. Type II Respiratory Failure with Compansated Respiratory Acidosis e. Type I Respiratory Failure with Compansated Respiratory Acidosis 142-Value of FEV1/FEC------>>>0.8 143-In Asthmatic patients what test will be less------>>>FEV1 144- A man with symptoms of Cushing disease------>>>Adrenal Adenoma 145-Best indicator of Renal clearance------>>>Inulin 146-Both GFR and RPF will be increased------>>>Dilation of afferent 147-A disease in which Antibodies against post-synaptic ACh receptors------>>> Mysthenia Gravis 148-A patient got injured and pupil reduced in size can't even dilate in dark------>>> Lesion to sympathetic Cervical trunk 149-Severe transfusion reaction ------>>> A+ to O+ 150-Atonic bladder lesion will ------>>> a. Loss of parasympathetic efferent (I did) b. Loss of somatic nerve supply 151-One Nasty Qx about "Voluntary" Contractions withO motor wil will be activated first a. Gamma motor will be activated first b. Both will be activated simultaneously (Ans) confirm it c. Stretch reflex is inhibited 152-Vitamin required for Fatty acid synthesis ----->>>Biotin 153-Type of fibers in Preganglionic Autonomic fibers----->>>B type 154-A needle piercing mid-axillary line will not damage a. Levator costrum (Ans) b. Internal intercostal muscle c. External intercostal muscle d. Innermost intercostal muscle 155-Which structure pass through Aortic hiatus----->>>Azygous vein 156-Farmer with linear foot lesion----->>>Cutaneous Larva Migrans 157-Somites a. Arise from both side of notochord (Ans) b. Arise from lateral plate mesoderm c. Arise from intermediate plate mesoderm 158-Intestinal motility is due to a. Na+ influx due to impulses b. Nervous stimulation of slow wave c. Spontanous stimulation of slow wave by Ca+ (Ans) BRS GIT Unit d. Hormonal stimulation of slow wave e. Simultaneous summation potential due to Na+ 159-In cardiac muscle pleatue is due to a. Influx of Na+ b. Influx of Na+ and Ca+ (Ans) c. Influx of Cl- d. Influx of K+ e. There was no option of K+/Ca+ influx Nor Ca ions influx So B near best 38

160-Na/K pump act by a. Pump Potassium Against the gradient (Ans) b. Throw 3K for 2Na c. When Ca+ increase intracellularly d. When Na + increase intracellularly 161-Chromatid align at equatorial plate----->>>Metaphase 162-Which Cells have telomerase a. Germ cells (I did) b. Endothelial cells c. Neutrophils d. RBCs e. Neurons 163-Regarding alveoli a. Surfactant avoid collapse of alveoli (Ans) b. Produce by type II pneumocytes c. Dec compliance 164-Mean arterial Pressure is------>>>Diastolic plus 1/3 pulse pressure 165-Dermal Artery damage. What compensatory mechanism will happen--->>> Vasoconstriction 166-Patient with DKA died. Which Oragism responsible---->>> Mucormycosis 167-Gastric HCL from parietal cells is needed for a. Conversion of pepsinogen to pepsin (I did) b. intrinsic factor activation 168-Perineal area innervation was asked------>>>S2-4 169-Most drugs are metablized in ------>>>Liver 170-Death due Clostridium botilinum---->>>Paralysis of respiratory muscle 171-Which drug cause HyperUrecemia------>>>Pyrazinamide 172-Hemorragic infarct in a. (Ans) b. Heart c. Spleen 173-About coronary circulation which is true a. Veins have similar names with arteries b. Arteries runs in respective atrioventricular grooves (True) c. occur at arteriolar level 174-Test for Typhoid fever in 6 days----->>>Blood culture (BASU) 175-True about Osmosis (I think 2 Options was true which are given) a. Is passive process b. Pressure depend upon Number of solute (I did) 176-Which of the following Aid to immune system to kill bacteria----->> a. Complement system (I did) b. Opsonins. {{This Qx was repeated in paper of all specialities on both 8th and 9th March}} 177-A man with no cough and fever CXR show diffuse lung fibrosis a. Smoking b. Organic dust c. Inorganic dust (Ans) d. Radon 178-If core temperature of body fall, body response will be a. Decrease BMR b. Shievring c. Anterior hypothalamus activation (Ans) 39

d. Vasodilation 179-Which drug cause Hypoglycemia most rapidly a. NPH (I did) b. Glibenclamide c. Metformin d. Insulin lente 180-41% Hematocrit means----->>>Composed of RBCs, WBCs and platelets 181-Spreading of tumor to other sites------>>>Destruction of E-Cadherin 182-Internal carotid artery a. Choroidal artery to lateral ventricles b. Run sidewall of cavernous sinus c. Anterior choroidal is its branch which supplies anterior and inferior temporal lobe (Ans) 183- True about Supernumary cervical Rib----->>>Compress brachial plexus and subclavian vessels 184-A female with blurred vision etc case of Multiple Sclerosis----->>>Demyelination of neurons 185-Node of Ranvier------>>> Lack of Myelin sheath 186-Qx about tuberculin test (Negative)----->>>Taking immunosuppressive drugs 187-Control of ventilatory drive under drastic dec in O2------>>>Apneustic centre in medulla (I did)??? 188-MCC of global blindness a. Bacterial infection (I did Rabia Ali Key) b. Viral infection c. Chlamydial infection (???) 189-Amylodosis in plasma Cells. What will appear in Urine----->>>light chain 190- I think one Qx of cell injury----->>>Glycine AA 191-Esophygeal phase of swallowing is affected in which disease---->>>Scleroderma 192-Thyroid Glands are enclosed in Which Fasica----->>>Pretracheal fascia 193-A 55 years old male with painless hematuria and Transitional cell carcinoma of bladder---->>>Tobacco Smoking 194-Sympathetic Overactivity cause------>>>Bronchial dilation 195- In lumbar puncture which structure will be damage----->>>ligamentum flavum 196-One Qx of Anencephaly----->>>Failure of Neural tube 197-Left Adrenal vein drain into----->>>Left Renal vein 198- A Qx of women with writing problem, site of lesion asked---->>>G.palidus 199-Tachycardia and Hipotension after Tonsillectomy (#PPQx) ----->>> Hypovolemic Shock 200-Which preserve body in state of shock "As a whole" ( famous Qx discussed on Page many times) ---->>>CNS ischemic Response thanks and all the best remember us in your prayers =-=-=-=-=-=-=-=-=-=-=-=-=-=-=-=-=-=-=-=-=-=-=-=-=-=-=-=-=-=-=-=-=-=-=-=-=-=-=-=-=-=-=-=-=-=-

40

(4)Surgery & Allied 7th March (Evening) by Dr.Hasnain Afzal (197)

Mix paper 1 n 2 evening session

Correct Answers Marked with Asterik (*)

I have marked most of the answers. But do not consider this final. There may be mistakes. Kindly read topics, and then follow what you think is best and seek senior’s guidance too. Thankyou.

1. Premalignant lesion A. Intradermal nevus B. Compound nevus.* C. Seborrheic keratosis

2. Regarding middle meningeal artery most appropriate.... A. Involved in extradural hemorrhage. (I marked) B. Can be fairly located outside the skull at pterion... C. Enters middle cranial fossa through foramen spinosum... D. Grooves inside of skull E. Supply supratentorial dura MOST EASY- STILL THEY MADE IT CONTROVERSIAL All are true

3. Minerals not found in or minerals with low bioavalibility A. Green Vegetables.. B. Cereals C. Pulses... D.tubers E. Roots

4. Medial boundary of ischiorectal fossa. A. External anal sphincter*... B. Levator ani. C. Ramus something(not sure)

5. Boy with h/o trauma to perineal region, swelling in scrotum and ant. Abd wall but didnt go to thigh. Rupture of.. A. Membranous urethra. .. B. Bulbar urethra*... C. Neck of urethra... D. Perineal body.. E.... 6. Turner syndrome... A. Short stature*. B. Autosomal dominant ...c. Autosomal recessive.

7. Pseudo hermaphrodite. With 46xy..... A. Adrenogenital synd.* 41

8. Abdominal aorta. a. Gives renal arteries at L2*. B. Lateral br. To parieties c. Present on the right of ivc...

9. Hepatic artery bile canaliculus and vein something . exact stem not remembered. a. Classic hepatic lobule b. Simple acinus .c. Compound acinus. C. Principal acinus

10. Stab to chest...air in pleural space will cause. a. Lung to collapse and chest wall to sprung out.*

11. Dead space doesnt change in A. Standing. B. Shallow breathing.*

12. Regarding 1st lumbrical. A. Arises from 2nd metacarpal b. Bipennate. C. Attaches to radial side of index finger. D. Attaches to proximal phalanx*

13. Lateral thenar muscles spared but other short muscles of hand paralysed. Injury? A. Ulnar*. B. Median .c. C5 c6. D. C8 t1

14 axillary artery. a. Lateral to pect minor. B. Post to pect minor*.c lat. To lat. Cord of brachial plexus..

15. Infection ant. To pretracheal fascia spread to . A. Ant. Mediastinum*. B. Sup. C. Post. D. Neck maybe. E. Middle

16. Sarcomas. A. Spread to bone. B. Clumps/clusters C. Radiosensitive D. Aggressive Read yourself- controversial

17. Primitve streak mesoderm migrates to anterior abdominal wall and around cloaca. A rupture of this mesoderm cranially will produce a.ectopic anal opening* b. Imperforate anus c. Rectovesical D. Bladder exstrophy. E. Hypospadias.

18. Hbeab +ve, hbcab +ve. Igm -ve. Hbsag -ve A. Acute resolving hepatitis.* B. Chr hep. ..repeated mcq from asim shoaib iguess.

19. Systemic fungal infection .drug given. A. Amphotericin* b. B.ketoconazole. 42

20. Vitamin a. A. Rhodopsin*. B. Collagen synthesis. Other options not remembered.

21. S2 in comparison to s1 sound. A. Inc. Frequency*. B. Longer duration. Other options.....???

22. B/l recurrent laryngeal nerve cut. Muscle spared. a. Cricothyroids*. B. All other arytenoids were in Other 4 options.

23. Superficial cardiac plexus. a. Contain both symp n parasympath*. B. Parasympath mainly. C. Contain fibers from all heart/vagal?? Nerves.

24. Gastric ulcer pain. A. Greater splanchnic*. B. Lesser splanc. C. Least splanch.

25. Appendicitis pain radiates to umblicus thru. A. T10*

26. H/o fall bladder incontinence unable to void urine. Spinal segment involved. a. S2s3s4.* B.s1ss2s3. C. L5s1s2.

27. Neck of fibula fracture. Dorsiflexion n eversion gone. a. Common peroneal n*.

28 excessive eversion lig. Ruptured. A. Deltoid.*

29. Person standing. Venous return from legs mainly facilitated thru. A. Venous valves. b. Contraction of sk. Muscles*. C. Other options...??

30. Medial side of arm stabbed. Flexion n supination affected. A. Musculocutaneous n.*

31. Mediolateral episiotomy. Structure cut. A. Bulbospongiosus muscle.* B. Levator ani muscle...c. .???

32. Regarding withdrawal reflex? A. Relaxation of flexor muscles B. Stretch of muscles(option not correctly remembered) C. Multisynaptic*

33. Tension in the muscle is limited by a.golgi tendon organ* 43

34. Alpha adrenergics causes. a. Mydriasis*. B. Bronchodilation. C. Lipolysis.

35. Due to stress of surgery inc peripheral something is due to secondary action from. a.acth*. B. Serotonin. C. Cytokines.

36. In crf unlikely A. Hypophosphatemia* 37. Opsonin. A. Fc igg and c3b* . B. Fab igg. C3b. C. Others like these options.

38. Tcell receptor complex bind with. A. Mhc*. B. Apc.

39. Gluteus medius n minimus. A. Abduct n medially rotate.*

40. Ciliary body . A. Pigment epithelium produces aqueous humor. b. Pigment epi of retina is cont. With pigment epi of ciliary body.* C. Non pigment epi produces aq. Humor.... D. Supplied by short ciliary arteries

41. Isthmus infront of. A. Rings 2 3 4*. B. Rings 3 4 . C. Rings 2 3

42. 4ml vial of 2percent lidocaine.. Total lidocaine. A. 80mg*. B.8mg. C.2mg.

43. 0.85percent nacl is equal to. A. 850mg/100ml* b. 85mg/1000ml...options not remembered Correctly.

44. St elevation v4. Artery involved. A. Lad.* B. Lt.circumflex. C. Rca.

45. Lt.circumflex artery involved in mi. Old mi with new mi. Area involved. A. Lt. Atrium n lt. Ventricle.*

46. Bundle of his blood supply. A. Rca*. B. Main lca. C. Interventricular br of lca. D. Marginal art. Controversial-Read yourself

47. Most commonly involved heart valve. A. Aortic. B. Mitral.* C. Tricuspid. D. Pulmonary.

48. Dka pt. Died. Which fungal inf. A. Mucurmycosis.* 44

49. Stroke death. Autopsy brain. Necrosis. A. Liquefactive.*

50. Abdominal . Artery involved. .A. SMA.*

51. 50 yrs old. Hypertensive. Severe chest pain radiating to back. Died. On autopsy findings. A. Medial necrosis of aorta*. B. Atherosclerosis

52. Median umblical lig is remnant of. A. Urachus.* B. Allantois

53. Premature baby born. Most common finding is A. PDA*. B. Patent foramen ovale

54. Pulmonary arteries supply. Part of lungs. A. Bronchioles. B. Alveoli*...other options..??

55. In adult head of femus blood supply. A. Obturator. B. Med n lat circumflex femoral art.*

56. During l.p structure punctured. A. Lig.flavum* 57. Return of blood during exercise from A. Calf muscles*

58. Testosterone main funtion. a. Grows hair on scalp. B. Inc muscle and bone growth*

59. Sertoli cells function. A androgen binding protein*. B. Testosterone production

60. Adult female breat atrophy. A. Estrogen*. B. Estrogen plus progesterone.

61. Carb protein fat synthesis hormone. a. Insulin*. B.cortisol. C. Thyroid.

62. Most common salivary gland tumor is of which salviary gland. a. Parotid*. B. Submandibular. C. Sublingual

63. Cervical ca. Lymph nodes involved. A. Internal iliac*. B. Superficial inguinal.

64. Regarding rectum. a. Starts at s1. B. 9 inches long. C. Also supplied by median sacral artery*. 45

65. Regarding anal canal. A.supplied by both sup n inf rectal arteries*. B. Inferior rect artery part of Portal system... Something like that. Options not remembered.

66. Pt. Started on oral anticoagulant. Test to monitor. A. PT.* B. Bt. C. Aptt

67. Coumadin (warfarin) question.. Options. A. Inc bleeding time. B inc coagulation time maybe.

68. Trachea commences at. C6.*

69. Vit k not absorbed. Coagulation factor decreased ...? a. Prothrombin*. B. factor VIII c. Options..????

70. Vit k def. First n foremost coagulation factor depleted. A. 7 b.2 c.10 d. Protein c.*

71. Female presents with dvts . Says she has been taking some kind of pills. a. Ocps.

72. 10 days fever. Chest pain unrelated to respiration. A. Myocardium* b. Pericardium

73. Chr. 9/22 a. Cml* b. Aml. C. Hodgkins.

74. Breast outer quadrant . A. Pectoral (anterior ) group*

75. Pregnant with jaudice. Went to remote village. To see her parents. Cause. A. Hepa b. Hep b. C. Hepc. D. Hepd. E. Hep e*

76. Liver transplant pt. I think died. Cause. a. Cmv*. B. Ebv. C. Hep a. D. Hep b. E. Hep d.

77. Which ca rarely metastasizes. A. Bcc*

78 bcc features. Most likely A. Locally malignant* 46

79.which ca spreads to large vessels. A. Clear cell ca of kidney*. B. Transitional cell ca of kidney . C. Lung ca... Other options not remembered.

80. Cholangicarcinoma.. A. Clonorchis sinensis.*

81. Most common ca of epiphysis. A. Osteoblastoma*. B. Osteoma. C. Ewings. D. Osteosarcoma. E.oteoid osteoma.

82.limb growth will be affected if fracture passes thru. A. Epiphyseal plate*. B. P.epiphyseal line. C. Metaphysis. D. Diaphysis.

83. Acoustic neuroma. A. Vestibular component of cn 8*. B. Cochlear part of cn 8. C. Trigeminal n. D. Facial.

84. Hypoglossal nrv injured. A. Tongue deviated to same side*. B. Other side. C. Not deviated.

85. Deep part of parotid fascia forms. A. Sphenomandibular lig. B. Stylomandibular lig*. C Buccopharyngeal membrane.

86. Intraarticular disc of tmj involved.. Muscle paralyzed. A. Lat.pterygoid*. B. Med pterygoid. C. Temporalis.

87. Lung ca fungiform growth. A. Bronchogenic ca. B. Small cell ca. C. Sq.cell ca. D. Undiff. Large Cell ca.

88. Gastric lymphoma ass with. A. H pylori gastritis.*

89. Terminal ileum resected more to 100 cm. A. Less absorption of bile salts*. B. More absorption of Bile acids. ..other options not remembered.

90. Femoral arterial pulse. A. Mid inguinal point*. B. Mid inguinal ligament. C. Adductor canal.

91. Inguinal canal. A. Absent in infants. B. Extends from ant.sup.iliac spine to pubic tubercle. C. Roof Formed by conjoined tendon*. 47

92. In. Old age . A. Vc decreases*. B. Co decreases.

93. Decrease in esr is due increase in . A. Fibrinogen. B. Albumin*.

94. Cytoskeleton connected to ecm environment thru. A. Intermediate fil. b. Proteoglycan*. C. Integrin. D. Cadherins

95. Glycocalyx contains. A. Carbohydrate moiety*. B. Lipid membrane...... options not clear.

96. Not electrical active transport of na occurs in. a. Pct*. B. Dct. C. Loops .d. Collecting duct.

97. Greatest clearance is of A. Pah*. B. Insulin-yes it was written insulin.

..

98. Rt. Testicular vein enters. a. Ivc. *

99. Tibial nerve severed. Which muscle will still flex the knee joint. A. Short head of biceps*. B. Long head biceps. C. Semitendinosus D. Semimembranosus

100.oraganelle with double membrane.

A. Nucleus.* B. Golgi. C. Rer. D.ribosome.

101. Bupivacaine. Overdose initial symptom. A. Unconsciousness. B. Vent arrhythmia. C. Ringing in ears*(I marked) Other options not remembered.

102. Pt. Anesthetized heat will not produce due to a. Loss of musle tone*. B. Na/k pump blockage.

103. 60 percent ventricle fill during .a. Rapid inflow*. B. Isovolumetric relaxation. C. Atrial contraction.

104. Regarding post. Comm artery. A. Connects PCA to ICA above occulomotor nrv*. B. Connect Pca below occulomotor nev. C. Connect ica to basilar artry. 48

105. Dorsal vagal nuclei present in. A. Medulla*. B. Upper pons. C. Lower pons. D. Midbrain.

106. Which hormone causes uncoupling of oxidative phosphorylation with increase heat production: a. Thyroid hormones* b. Insulin c. Growth hormone d. Epinephrine

107. Patient had previous episodes of diplopia nd dizziness now acutely comatose reason... A. Basilar artery thrombosis*, b .c .d.3 type of hemorrhages were in options, e .carotid artery occlusion

108. Child has episodes of bleeding nd echymosis i guess since birth hb 9.8 mg/dl nd platelets count was normal 180 x 10^3 bt prolonged... A. Hemophilia, b.itp c.,platelet functional defect*

109. Patient given warfarin on second day had large echymosis on thigh... a. Ttp B. Overdose of warfarin C. Protein c def*

110. Csf has low conc of which of the following than plasma... A. Osmolarity, B. Na, C. K+, D. Mg2+ E.proteins*

111. Corona radiata forms from.

A. Germ cells. B.theca interna. C. Theca externa d. Granulosa*

112. Surgery grade 3 tumor.

A) more than 75% cells undifferentiated

B) 50 to 75 % cells undifferentiated* (i marked)

C. 100%

D. 25% or less i guess

113. Muscle attached to bone by. A. Tendon*. B. Ligament.

114.treponyma pallidum for intial diagnosis sample can be taken form? A. Genital sore scrapping* 49

115. Pt with jaundice hepatosplenomegaly....complete stem not remembered. Which test to be Performed. A. Antimitochondrial ab*

116. 1 litre blood loss quest. A. Inc venous tone*

117. While playing cricket got abrasions. Immediate mechanism. A. Vasoconstriction* Repeated mcq.

118. Highest diffusion both across resp membrane and body fluids. a. O2. B. Co2 C. Helium. D CO E. Nitrogen

118. Tumor supressor gene. A. P53*

119. Mc cause of fatty liver in our population. A. Hep B & C* B. Alcoholism

120. Ecf volume can be increased by. A. Hypertonic saline*. B. Hypotonic saline. C. Distilled water.

121. Most radiosensitive tumor. A. Seminoma* B. Glioma.

122. Alveolar surface dry due to. A. Surfactant. B. Inflammatory cells. C. Negative interstitial pressure* (i marked) controversial- better to read topic

123. Pseudomonas aureginosa. ...complete stem??? A. Endotoxin*.b. Exotoxin.

124. Corynebacterium diphtheria lethal effect on? A. Tonsil.b. Heart*. C. Kidney.

125. Benign tumor. A. Leiomyoma*.

126. Histoplasmosis effect on . A. Reticuloendothelial sys*. 50

127. Highest na. Iguess. A. Node of ranveir*. B. Axon. C. In the myelin.

128. Rectus sheath post.lower limit A. Arcuate line*. B. Pectineal lig or line. C. Lacunar.

129. Bioavailability is not affected by? A. Sex/gender*

130. Regardig morphine use. A. Ternimal cancer pt.*

131. Heparin prevent. A. Propagation*. B. Embolization. C. Dissolution. D. Organification.

132.upper motor neuron lesion?? A.fasiculation b. Increase muscle tone* c. Dec tendon reflexes

133.passes throug cavernous sinus A. Abducent*

134.young girl,used a drug for back ache, platelets were in normal range,wbc 7000,neutropenia, hb was 9, small imature cell in peripheral film???

A.aplastic anemia

B.acute leukemia*

C.itp

D.analgesic induced (i marked)

135.difference btween arterial nd venous blood??

A. Packed cell volume* b. PH.

136.gastric emptying accelerated

A.gastrin*

B.secritin

137.in juxta glomerular nephron,sodium is activaly reabsorbed electrically neutral from lumen of nephron in which segmnt of nephron??? a. Pct.* B. Dct. C. Loops.

138.about follicular ca of thyroid???

A. Psamoma bodies 51

B. Vascular invasion*

139.transplanted auto graft survives most in A. Uremia B. Agamaglobinemia*... C. third degree burns ,

140 local edema

A.allergy*

141.increase in creatinine,,source??

A. Liver

B. Skelatal muscle*

142.gametogenesis,,one cell get extra than normal haploid???

A. Trisomy*

B. Monosomy

C. Non disjunction

143.chronic hepatitis histology

A.lymphocytes b. Fibrosis*

144.strongest layer- A. mucosa, b. Sub mucosa* c. Longitudinal layer d. Circular layer

145. Right atrium wala question was ,,,if presure in right atrium increases with in normal limits what Would happen A. Hr inc* B. cardiac output inc C. Decrease venous tone (options not correctly remembered)

146. Weight bearing line in pelvis pass thru??

A.ischial tuberosity*

B. Coccyx.

C. Pubic ramus 52

147.sugar in urine will be detected if its concentration in blood becomes

A. 250mg/dl

B. 200mg/dl* (according to f.a)

C. 180 mg/dl

D. 150 mg/dl

E. 375mg/dl

148. Which of the following is ectoparasite.

A. Lice*

B. Fleas

C. Ticks

D. Bed bugs

149. Farmer organophosphate poisoning what should be given. A. Atropine* b. Pralidoxime

150. For chemotaxis. Exact stem not remembered. A. Lymphokones* .b.serotonin

151. Baroreceptors responds to? A. Dec blood pressure*. Complete options not remembered.

152. All preganglionics neurotrasmitter A. Acetylcholine*

153. Sweat gland by. a.sympathetic*

154. Rt upper n lower limb paralysis. Angle of mouth deviates to left while talking. A. Internal capsule* B. Pons. C.cortex

155.sulfur containing aminoacid A. Cystine*

156. Truncal vagotomy A. Dec acid secretion and pepsin* 53

157. Insulin is inhibited by. A. B.blockers* B. Secretin C. Glucagon D.b.agonists

158. Primary active transport... A. Enzymes b. Carriers c. Proteins d. Pumps*

159. Non bacterial thrombotic endocarditis A. Terminal cancer* .b sle

160.mid arm circumferece. A. Proteins* b. Fat. Options and stem not clearly remembered

161. Injury to pulmonary alveoli causing constriction of alveoli by a.hypoxia b.atherosclerosis c.adenosine d.prolonged oxygen therapy*

162. On autopsy finding... Complete stem required... a. Hypertrophied parathyroid gland*

163. Boy. H/o taking antimalarials. A. G6pd def*

164. Chloride shift in rbc

A. Ankyrin B. Spectrin C. Band 3*

165. Ppd false negative A. Malnutrition B. Immunosuppressant*

166. Fisherman bleeding gums A. Scurvy*

167. Mcq related to wilsons disease

A. Ceruloplasmin*

168. Following lymphoid organ recive lymph from subscapular sinus a.thymus b.spleen c. Lymph node* 54

169. Adrenectomy taste preferences? A. Nacl* 170. Most important cell of chronic inflammation A. Lymphocytes B. Macrophages*

171. Test for typhoid in ist week... A. Blood culture* 172.patient with lung abscess develops meningitis, causative organism.? A. Staphylococcus*

173. Question regarding edema--- A. Lymphatic blockage*

174. 41% hematocrit... A rbcs only* b. Rbcs wbcs, platelets (i marked) -kindly confirm it by yourself-

In ganong’s physio “the number of red cells is also conveniently expressed as the hematocrit , or the percentage of the blood, by volume, that is occupied by erythrocytes”

175. Low dose estrogen pill causes..... A.breast cancer..b.dvt...c.uterine cancer...d.cervical cancer e. Hepatic adenoma*

176. Human is an intermediate host of A. Hydatid cyst*

177. Severe transfusion rxn. a. Apos to oneg* B. O to bpos. C. Apos to ab pos. (behavioral sciences/c.med questions…) 178. 2x2 table A. T test. B. Anova . C. Chi square.

179. Correlation of two things by which test A. Student t-test B. Anova test C. Mc witneys D. Chi square

180. How can doctor provide better management to patients: a. By better communication * 181. Another behavioral sciences question. (see rabia ali mcqs b.sciences section ) Ans by active listening to patient.* 55

182 eeg of a patient shows A. Increase frequency of waves in active mind than in deep sleep B. Is symmetrical in both hemispheres C. Small amplitude of waves in intense thinking D. Determine intelligence

183. Regarding eeg...

184. .sodium regulation is through

A osmoreceptors* B .sodium absorption C .adh D .reninangiotensin E .aldosteron

185. Teratogenic effect in which embryonic age? A. 2 to 8 wks* B. 1 to 3 wks C. After 12 wks

186. Which form of thyroxin is an active form A. Reverse t3 B. Unbound form* C. Bound to albumin D. Bound to prealbumin

187. Beta receptor stimulatin causes A. Increase heart rate* B. Increase force of contraction C. increase renin from kidney D. increase lipolysis

188. Drug interaction occur A. With drug only B. Pharmacodynamic only C. Pharmacokinetic only D. Pharmacodynamic and Pharmacokindetic Both*

189. Parasympathetic effect on Heart-A. Increase PR interval*

190. DIC activated by?--A. Tissue thromboplastin*

191. Regarding PseudoHYPERparathyroidism? A. because of Hypocalcemia B. Because of PTHrP by various tumor* 56

C. associated with Nephritic syndrome D. because of Vit D def

192. which of the following augment the effect of immune system? A. Complement System* B.opsonins Other options not remembered

193. For sodium neutralization A. K+ B. organic ions c. Cl d. Mg e. HCO3

194. Regarding occupational diseases increase risk of problems with exposure to: a. coalmines b. asbestos* c. silicons d. textile industry

195. Regarding biceps brachi a. insertion on humerus b. Arises from supraglenoid tubercle* c. pronation d. extention of elbow

196. Inf wall MI:--A.Right marginal artery*

197..A person had an accident and there was severe bleeding from his body.He got multiple transfusions but Ecg showed Changes after transfusion due to a hypocalcemia b hyperkalemia* c hypokalemia d hypermagnesemia e PH changes

=-=-=-=-=-=-=-=-=-=-=-=-=-=-=-=-=-=-=-=-=-=-=-=-=-=-=-=-=-=-=-=-=-=-=-=-=-=-=-=-=-=-=-=-=-=-

57

( 5)-Surgery & Allied 7th March Evening-by Xaheer Khan (185 MCQS) 1.Regarding abdominal angina which artery is involved a..inf mesentericartery b..aorta c..sup mesenteric artery (answer) 2...... 50 years old hypertensive patient presented with severe chest pain radiating to back suddenly collased On autopsy what can be seen a..ischemic necrosis b.medial necrosis (answer) c.athero sclerosis 3....Medial umbilical ligament is the remnant of a..umbilical artery b..umbilical vein c.alantoise d..urachus (answer) 4...... Pulmonary artery supplies a.alveoli (answer) b.bronchioles c.terminal broncioles 5...... premature baby born with a..PDA (answer) b.patent foramen ovale c.right atrial pressure increases 6...... 2×2table...... chi square test 7...... Cause of adult female atrophy.....Estrogen 8.....Which harmone causes fatty acid synthesis and protein synthesis a.cortsole b.thyroid c.glucagon d..insuline(answer) 9.Most common salivary gland.... a.salivary b.submandibular c.parotid(answer) 10...... Lymph nodes invol vein Ca of cervix... Internal iliac nodes (answer) 11.....Regarding rectum a.startsatS1 b.9incheslength c.has two lateral curves d.covered by peritoneum all around e.also supplied by median sacral artery(asim key) (answer) 12...... Regarding anal canal a.supplied by both sup n inf rectal artereis (answer) b.inf rectal artery part of portalsystem c.external anal sphincter is involuntary d.ha stwo lateral curves e.sup inguinal lymph nodes drain the upper half of anal canal(asim key) 13.....Patient started on anti coagulant how to moniter further a.Aptt 58

bClottingtime c.Pt (answer)

14.....Trachea commences at....C6 15....First clotting factor decreased in vitK deficiency a.factor8 bfactor5 cfactors d.Factor2 (answer) 16...... Afemale of reproductive age presented with dvt.she gave history Of taking some kind of pill what is the cause of her condition a.thrombo embolism b.analgesics c..OCPs (answer) 17...... 45 years old man presented with chest pain not related to respiration myocardium (answer) 18.....9:22 translocation of grnesin CML 19.....A fisherman presented with echymosis n bleeding from gum she is Mostlikely suffering from a.Vit c def (answer) b.Vitb12def cFolic acid def d.Iron def 20...... A pregnant lady from remote village presented with Mildly raised sgpt n bilirubin from 2 months what can be the cause a.Hepd bHepb cHepe d.Hep a (answer) 21.....A patient with liver transplantation there is increase risk of problems from a.ebvvirus b.CMV (answer) c.Hepd dHepaande 22...... which cancer rarely metastasizes Basal cell carcinoma (answer) 23...... The features of BCC a.metastasis b.beningtumor c.bordersareundermined d.Locally malignant(asim key) (answer) 24....cancer spreading to large vessels a.Clear cell ca of kidneys (answer) b.lungca c.transitional cell ca (asim key) 25...... Acoustic neuroma invoving a..cochlearpartof 8thnerve b.vestibular part of 8thnerve (answer) c.trigeminalnerve d.fascialnerve 59

26...... Epiphysial bone tumor a.osteoidosteoma b.osteosarcoma c.ewingssarcoma d.Osteoblastoma (answer) ●osteoblastoma is a large group of bone tumor which also consists of ostepidosteoma.they all involve epiphysis.They are rare tumors.The most common is Giant cell ca which was not in the options●

27...... Hypoglossal nerve paralysis can lead to Tongue is deviated towards the leision site (answer) 28....Deep part of the parotid fascia forms a.bucco pharyngeal membrane b.stylomandibular lig c.sheno mandibular lig (answer) d.stylo hyoid lig 29.....Intra articular disc of TM JOINTgvs origin to a.medialpterygoidmuscle b.temporalis c.buccinator d.lateral pterygoid muscle (answer) 30...... a 30years old heavysmoker presented with emptysis.On xray film there is a fungating mass.Diagnosis a.squamous cellca (answer) bsmallcellca c.bronchogenicca d.mesothelioma 31...... Gastric lymphoma most probable cause a.Hpylori (answer) b.chronicgastritis c.duodenalulcer d.smoking e.analgesicuse 32.....first Lumbrical a.bipennate b.attached to proximal phalynx c.origin near the radial side off first meta carpal bone (answer) ●First n second are unipennate while third n fourth are bipennate● 33....Ina surgery1 00 cms of ileum was resected. what will be the outcome a.les absorption of bile salts (answer) b.less absorption of bile acids c.more absorption of bile salts d.increase sodium loss 34.....Regarding inguinal canal a.extends from sup iliac spine to pubic tubercle b.roof is formed by conjoint tendon (answer) c.lowerborderformedbyfasciatransversalis 35...... In old age a.cariacoutputincreases b.venousreturnincreases c.Frcincreases 60

d.VC decreases (answer) 36...... decrease ESR is due to Increase albumin (answer) 37...... Cytoskeleton connected to ECM through a.intermediatefilaments b.cadherins c.proteoglycans d.integrins (answer) ●principal role of integrins is connection of cytoskeleton to extracellular matrix Wheras e cadherins bind to both ecm n icm● 38....Neutral electric transport of Na occurs in a.DcT b.loopofhenle c Pct (answer) d.collecting ducts 39...... Greatest clearance is of PAH (answer) 40....Regarding cardiac plexus a.formed by both symathetic n parasympathetic nerves (answer) b.thoracic splanchnic nerves are the main nerves c.starts at trachea bifurcation d.present at the base of heart e.anterior cardiac nerves are the main nerves 41....Baroreceptors a.hypotention b.respond to progressively increasing arteriolar pressure (answer) c.respondsto progressively decreasing arteriolar pressure 42....Insuline is inhibited by a.secretin bglycogen cbetaagonists d betablockers (answer) 43.....Glycocalyx contains A carbs moities(back bone of glycocalyx) (answer) b.phospholipids c.antigens d.proteins 44....Biaoavailibity of minerals is lowest in a.roots b.green leaves c cereals d pulses e.Tubers (answer) ●All others must have mineral sexcepttuberehich have less.Tubers are potatoe family rich in starch● 45....Drug drug interaction by Both pharmacokinetics and pharmacodynamics (answer) 46....Glucose start appearing in urine at a.100mg b180mg c200mg 61

d.350mg e 250mg (answer) ●glycosuria....morethan200mg● 47....a patient suffering from lobar pneumonia causative organ a.staphylococci (answer) bstreptococci c.pneumococci 48...correlation of two things by which test a.studentt test b anova test c mc witneys d.chi squared make it clear..i did chi square 49....Regarding sweat glands Cuboidal epithelium(ppq) (answer) 50....Bioavailibility is not affected by a.age b.liverdisease c sex/gender (answer) 51...... The alveoli are kept dry by a.macrophages b.negative intra pleural pressure c.ventilation d.surfactant (answer) 52...... The dorsal vagal nucleus is presentin a.medullaoblongata (answer) b.midbrain cpons 53...... A person present with diplopia and dizziness for the past one month suddenly went into comma the cause lies in a.cerebellar haemmorhage b.pontine haemmorhage (answer) c.thallamic hmg d.subarachnoidhmg 54...... muscles are attached to the bones by a.ligaments b.aponeurosis c.fasciae d.tendons (answer) 55...... Dimercaprol a.increases clotting b.decreases clotting (answer) c.decreases bleedingtime 56...... A young lady presented with increase bleeding time.It is due to a.vonwilli brand factor (answer) b.factor8 c.factor9 d.itp e.platelet dysfunction 57...... For sodium neutralization a..k 62

b.organicions c.Cl (answer) d.mg e.hco3 58...... Amino acid containing sulphur Cystein (answer) 59...... Sodium regulationis through A osmoreceptors (answer) b.sodium absorption c.Adh d.Reninangiotensin e.Aldosteron 60...... Circumflex artery supplies a.antsurfaceofheart b.left atrium n ventricle (answer) c.rightatrium d.leftatrium e.leftventricle 61...... 41%haematocrit contains a.rbcs b.rbcs,wbcs,platelets (answer) c.platelets...... ganongsbcq 62...... arterial blood as comparedto venous blood has a.lowph b.more packed cell volume (answer) c.increasepo2 d.increase ph...(asim key) 63...... During Lp which structure is ruptures...... ligamentum flavum (answer) 64...... during stab injury Ipsilateral lung collapse n ipsilateral chest expands (repeated question) (answer) 65...... a person got lacerated injury on his elbow what will be the initial affect of blood haemostasis vasoconstriction....rmem vessels on the top always (answer) 66...... during a one km running the venousreturn to the heart is by a.valvesinthelegveins b.pumpsinthelegs c.skeletal muscles contraction (answer) 67...... right atrial pressure is increased this will lead to a.inc cardiac output (answer) bheartrateinc c.inccontractionofrightventricle 68...... regarding occupational diseases increase risk of problems with exposure to a.coalmines b.asbestos (answer) c.silicons d.textile industry 69...... Eversion of foot leads to ligamentum fractured a.laterligament b.planatarisruptured c.Deltoid ligamentum (answer) 70.....Most common causeof fatty liver in our society is 63

HepB and HepC (answer) 71...... whichnerveleavesthethenarmusclesandallothermusclesofthehandareaffected.Thenerveinj uryisto a.ulnar nerve (answer) b.median nerve c.c8t1 d.Radial nerve...asim key ●in this question thenar muscles are spared.The answer is not c8t1 because e all the muscles of the hand are supplied by this root N here all muscles are not affected● 72...... A farmer while spraying suddenly got collapsed with salivation from his mouth n broncho spasm.What is the specific antidote a.atropine b.neostigmine cphysostigmine d.Pralidoxime....asim key (answer) 73...... Seventy years oldmale develops sudden loss of power in both right upper n lower limb with angle of mouth deviated to the Left during talking.The most likely site of leision is in a.internal capsule asim key (answer) b.pons c.medullaoblongata d.midbrain 74...... Larynx lost its supply but its still can be contracted due to the nerve supply to a.thyrohyoid b.postcrici thyroid c.cricothyroid (answer) d.cricoarytenoid 75...... Regarding biceps brachi a.insertion on humerus b.supplied by musculo cutaneous nerve (answer) c.pronation d.extentionofelbow 76...... blood supply in an adult to head of femure gets its branch from A lateral n medial circumflex femoral (answer) B arcuate artery C abturator artery d inf gluteal e supepigastric....retinacular wasn’t ino ptions 77...... A person was walking bare feet suddenly stepped upon some sharp object.he quickly removed his leg due to which reflex a.flexor response b.mono synaptic reflex c.multi synapticreflex (answer) 78...... Regarding general anesthesia body temp decreases due to which mechanism a.dec na k pump b.dec skeletal muscle tone (answer) c.stimulated by post hypothalamus 79...... Nucleus anmbiguos is in a.upper midbrain b.lower midbrain 64

c.medulla (answer) d.pons 80...... inadolcentthegrowthoflongbonessuddenlystoppedduetoleisionin A epiphyseal plate (answer) B metaphysis C epiphysealline D diaphysis 81...... low estrogen used by a 35 years old married femal.there is increase risk of a.liver adenoma asim key (answer) bthromboembolism ccabreast dcacervix 82...... action of gluteus medius and gluteus minimus Abduction and medial rotation (answer) 83...... Opsonization due to interaction of FCportionofigGandC3b (answer) 84...... skinreplacementpatchissuccessfulin a.neoplasm b.burnpatients c.autoimmunity d.aggamaglobunimia (answer) 85...... Morphinisusedin terminalcancerillness (answer) 86...... Aorta a.bifurcatesatL5 b.bifurcatesatT5 cliesattherightsideofIVC d.GivesrenalarteriesatL2 (answer) 87...... InfwallMI Right marginal artery cpsp demo quest (answer) 88...... Bundle of hiss is suppliedby a.LCA bRCA c.leftascendingbranchofLCA d.interventricular branch of LCA (answer) ●itsacontroversialqueation.IhvalotdiscussionregardingdiswdseniorsbutstillitsindoubttheysayRCA nmedscapenkaplan saysLCAalsoasim keysaysLCA.sodoitonurownrisk.ididwhatmymindtoldme● 89...... double membrane bounded organnel Golgi apparatus..endoplasmic reticulum..nucleus (answer) these all were given now we dnt know wats cpsp key (answer) 90...... Regarding MMA a.pterionis the surface marking b.its branches form grooves on the under surface of skull c.cause extradural hmg wen ruptures (answer) d.enters skull thru foramen spinosum ●all the options were correct.And was asked about most appropriate.i did extradural hmg due to its specificity most common adversity● 91...... A female delivered a baby and she was started on warfarin as she developed dvt.The next day she has a purplish patch on Her right thight what can be the cause 65

A lupus anticoagulant (answer) b.warfarin overdose c.proteinc def d.proteins def 92...... Frst factor affected in anticoagulation factor7 (answer) 93...... Bupivacain first sign of over doze a.arrythmias b-ringingine ars (answer) c.hypotention d.death e.decco ●it was asked initial affect so Cns excitation is the initia affects followed by CNs depression● 94...... V4stelevationMidue LAD (answer) 95.....vagotomy done for a.decmotilityofgut b.decgastricacidity c.decpepsin d.dec gastric acidity n pepsin motility of gut (answer) 96...... pain of Gastric ulcer is due Greater splanchnic nerve involvement (answer) 97...... medial wall of ischiorectal fossa is formed by Levator ani muscles (answer) 98...... most radio sensitive tumoris seminoma》》glialglioma》》craniopharyngioma (answer) 99...... Man is intermediate host in Hydatid cyst 100...... a person got fracture of neck of fibula now he cant do dors flexion n eversion Common peroneal nerve (answer) 101.....a young man fell from two storey building and now he has urinary incotinence.leision is at s2s3s4 (answer) 102.....s2 differs from s1 by a..s2hasincduration b.s2hasdecpitch c s2 has inc frequency (answer) ds1hasincpitch es1hasdecduration 103...... a child presented to opd with 104 fever for th e last 4 days.He has been passing cola colored urine for the last one day.He has bee n on anti malarials too.His lfts are deranged wdincunconjugatedbilirubin.What is the most probable diagnosis a.druginducedjaundice b.blackwaterfever c.paroxysmalnocturnalhaemaglobinuria dhaemolyticuremicsyndrome e G6pd (answer) 104...... post communicating artery connects Internal carotid artery with post cerebral artery (answer) 105...... ayoungmangotsomeinjuryandafterthattherewasswellingindascrotum followedbyantabdominalwallswelling 66

buttherewasstrictdemarcationtothethigh.Whichstructurewasinjured a.collesfascia b.bulbar urethra (answer) c.ejaculatoryduct d.penileurethra e.pelvicdiaphragm 106...... pseudo hermaphrodite with 46XX karyotype in Aadrenogenital syndrome (answer) b.turnorssyndrome c.klenifelterssyndrome d.45xo 107...... Turnor syndrome with a.AR b.AD c shows secondary infertility d stunted growth (answer) 108...... ECF is increased by infusion of A hypotonic saline bnormasaline chypertonic saline (answer) d dextrose water 109...... peri capsular sinuses in a.lymph node (answer) b thymus c palatine tonsils 110...... weight bearing line passes through Ischial tuberosities (answer) 111...... A14 years old girl presented with right iliac fossa pain.pain associated with vomiting and was radiated to periumbilical region.Which segmen is involved/or pain radiation due to a. ilioinguinal nerve b.iliohypogastric nerve c.t12level d L2level e T10 level (answer) 112...... microscopic slide of liver shows portal vein hepatic artery and bile duct at periphery and central vein in the centre.What is This called a.hepatic lobule b portal acinus c heatic acinus (also called liver acinus) (answer) 113...... EEG of a patient shows A increase frequency of waves in active mind than in deep sleep (answer) B is symmetrical in both hemispheres C small amplitude of waves in intense thinking D determine intelligence 114...... what is produced in muscles A phospholipids B creatinine (answer) C urea D uricacid 67

E glycogen 115...... Cervical lyph nodes drain into Internal iliac nodes (answer) 116...... about sertoli cells A produce testosterons B rapidly deviding cells C release androgen inhibiting factor (INHIBIN) (answer) D break blood testis barrier E reinkes crystals 117...... histoplasmosis whic sytem affected mostly A cvs b git c cns d reticulo endothelial system (answer) 118...... in CRF whatcantbepresent A lowph b hypo phasphatemia (answer) c enlargedparathyroids d metabolicacidosis ehypocalcemia 119...... on autopsy of CRFpatient which organ was hypertrophied Para thyroid glands (answer) 120...... strongest layerof smallbowel asubmucosa (answer) bmusoca clongitudnallayer dcircularlayer elaminapropria 121...... chronic hepatitis histology on microscope shows A fibrosis b neutrophills c lymphocytes asim key (answer) dlymphokines ●remember the pre dominant feature of chronic inflammations is LYMPHOCYTES● 122...... Dilute tubular fliud in which segmentof nephron A thick ascending limb (answer) B thindecending limb c distaltu bule d macula densa e collecting ducts 123...... sarcoma is differentiated from carcinoma A metastasis B aggressive (answer) C radiosensitive ●SOME says mets but mets are in both but with sarcoma through blood and carcinoma thorough lymph nodes.Sarcoma involves Large areas wd increase necros is increase haemmorhages n mostly creul to therapy● 124...... systemic antifungal is Amphoterecin (answer) 125...... heparin inhibits A organization of clot 68

B retraction of cot C dissolution of clot D propagation of clot.....asim key (answer) 126...... During pregnancy which period affects fetus mostly A 1to2weeks B 2to8weeks (answer) C 22to36weeks D last trimester E 7to12weeks (organogenes is from 3 to8 week so b suits da best) 127...... muscle is prevented from tearing under action of which receptor a.muscle spindles b GTO (answer) c ruffinis d meissners e phosphate bonds 128...... max sodium entry in nerve fibre takes place at A during depolarization B during repolarization C at nodes of Ranvier (answer) D as the impulse propagates along lenth of nerve fibre 129..... Regarding Axillary artery A it lies post to pectoralis minor muscle (answer) B lateralchordliesposttoit C startsatthelateralborderof pectoralis minor D all the chords are lateral to it 130...... Primary Brain vesicleis Mesen cephalon (answer) 131....0.85%normalsalinecontains a.850mg/100ml b 8.5mg/100ml c 8.5mg/1lit d.85gm/1lit e 85mg/100ml (idid option B but iam not sure about its answer.aplogies) 132...... the valve which has more propensity toward diseases is MITRALVALVE (answer) 133...... marranticendocarditisleadingtomitralstenosisisdueto A rheumaticfever B SLE C streptococcal glumerulonefritis D thrombo embolism E neoplastic diseases (answer) ●marranticisanoldnamefornonbacterialendocarditis.Marrantic means wasting away...so dats why it is strongly related to Diseases causing wasting■■cancers■■although also due to sle but neoplasms have stong adherance● 134...... A young lady came to ur clinic.How can u treat herina best way A good medicalknowledge B beneficial lab tests C clear conversation (answer) 69

135...... 23 years old boy came with mild jaundice.His hbsAg is negative...anti Hbc igM is neg...... anti Hbe positive.Whats the current situation a acute resolving phase (answer) b acutehepatitis c window period d chronic hepatitis 136...... difference between csf and cerebra capillaryis a.incph b carbohydrates c incpco2 d less proteins(they are almost negligible in csf) (answer) 137...... Lower limit of rectus sheath posteriorly is Arcuateline (answer) 138...... mostbeningtumorofsmoothmusclesis Leiomyoma (answer) 139...... pseudomonas producing shock through A TSST b catalase c endotoxins (answer) d exotoxins e lymphokines 140.....Highest diffusing capacity across respiratory membrane and in bodyfluids is A helium b carbonmonooxide c carbondioxide (answer) d nitrogen e oxygen 141...... Tumor suppresser gene-p53 142...... upper motor neuron leision leads to A dectendonreflexes B fasiculations C inc muscle tone (answer) D flaccidparalysis 143...... which nerve passes through cavernous sinus -6th nerve 144...... premalignant growth A sebborhickeratosis B compoundnevus...asim key (answer) C intradermalnevus D dysplasia e junctionalnevus 145...... 35yearsoldgirlhasbeenusinganalgesicforbackache.Shepresentedwithplateletsinnormal range.wbc7000 neutropenia.Herhbwas9withsmallimmaturecellsinperipheralsmear.Diagnosis a acute leukemia (answer) b ITP c aplasticanemia d druginduced 146...... vitKdeficiencyleadstodec A proteins B proteinc C factor8 70

D prothrombin (answer) 147...... Gastric emptyng increasesby a.secretin b.Gastrin (answer) c.Cck d.ach inhibition 148...... Grade 3 means a.20%undifferenciated b.30to50%undiff c. 50to70%undiff (answer) d. morethan70%undiff e .100%undiff 149...... local edema cause-Allergy (answer) 150...... Exogenous antigens..(stem nt clearly remembered but answer was MHC) 151.....most lethal effect of Diphtheria is on A cns b neurons c heart (answer) d lungs 152... During meiosis abnormalities may occur.If one of the cell get san extra chromosome instead of haploid number.The Condition is called A non disjunction B monosomy C robertsonian dislocation D Trisomy (answer) 153...... a lady presented with yellow eye s and she has got hepato megally.Alsoxanthomas present.What can be the cause in her A AMA (answer) B ANA C sclerosing cholangitis D lipidemia 154.....pseudo hyperparathyroisim A parathyroidharmone B pthrp (answer) c.hypocalcemia 155...... Right ovarian artery drains into IVC 156...... ciliary muscle supplied by –short ciliarynerves (answer) 157...... Biceps brachi attached to -Supra glenoid tubercle (answer) 158...... Which is ecto parasite- A Lice (answer) B tics C fleas D leech E mites 159...... Breast outer quadrant lymphatic drainage by A post B supraclavicular C lateral D anterior(pectoral) (answer) 160...... A 22years old man came to Opd with ALT raised viral marker s were negative.He has got some problem in the eye and Immunological tests were derranged.Whats the most useful investigation for him A liverbiopsy B HbsAg cHCVvirology d serum copperlevel e cerulo plasmin level (answer) 161...... adiabeticpatientpresentedwithincreasingdegreeofprogressiveillnessinvolvingrespiratory system.gastrointestinal.What can be the cause a histoplasma b acidbasebalance c Mucormycosis (answer) d hyperkalemia e bloodglucose 71

162...... A person got an accident and there was h severe bleeding from his body.He got multiple transfusios but Ecg showed C hangesaftertransfusiondueto A hypocalcemia B .hyperkalemia (answer) C hypokalemia D hypermagnesemia E PHchanges 163...... DIC activatedby A lowerythropoitine B endothelialdamage c. thromboplastine (answer) d clottingfactors e septicemia 164...... Cardiac plexus has vagus nerve supplyfrom which nerve nucleus medullaoblongata (answer) 165...... one question was regarding rhodopsin answer was VitA 166...... Doctor patient relationship Active listening (answer) 167...... lidocain vial contains 4ml normal saline of2% a2mg b800mg c4mg d.8mg (answer) 168...... isthmus lies at 234 tracheal rings (answer) 169...... after adrenelectomy taste preference increases for A sweets b.Nacl (answer) c sour 170...... 60%of ventricular filling is because of A Rapid inflow asim (answer) B slow inflow C diathesis D slowejectionphase E isovolumetricrelaxation 171...... about follicular cancer of thyroid a.vascular invasion (answer) b psammomabodies c lymphaticspread d hyperchromatism 172...... proteinon Rbc membrane for chloride exchane a.spectrin b.protein band3 (answer) c.glycocalyx 173...... Regarding cholangio carcinoma chlonorcissinensis (answer) 174...... treponema pallidum initial diagnosis amouthulcer b.genital sore (answer) c.tabesdorsalis 72

175...... Dead space doesnt decrease in a.shallow breathing (answer) b.deep breathing c.lung diseases d.gravity e.tracheostomy 176...... necrosis of Brain Liquefactive necrosis (answer) 177...... chemical mediator of inflammation a.histamine (answer) b.serotonine c.lymphokines 178...... a young. Man while walking most of cardiac output is due to Skeletal muscle contraction (in both papers) (answer) 179...... Adverse blood transfusion reaction occurs in A o+toA+ B onegtoA+ C AnegtoA+ d. AB+toA+ (answer) e AbnegtoA+ 180...... regarding choroid Choroid is continue with the pigment layer of retina (answer) 181...... Regarding un coupling of Oxygen that’s produced by a.thyroid (answer) b.norepinephrine c.epiephrine 182...... If a 70kg healthy adult loses one litre ofblood within 5 minutes there would be a.decTPR bi.ncGFR c.inc venous tone (answer) d.dec diastolic pressure e.splanchnic vasodilatation 183...... injury to pulmonary alveoli causing constriction of alveoli by a.hypoxia b.atherosclerosis c.adenosine d.prolonged oxygen therapy (answer) l84.....Primitive streakmesoderm cells migrate Round cloaca lmembranetoform antabdominal wall cells.If this migration doesnt happen around cloacal membrane will rupture and open cranially as a extrophyofbladder b imperforateanus c ectopi canal opening (answer) d hypospadias 185...... one was that sum flexion can still be maintained after nerve injury Answe was short head of Biceps(dnt remember clearly) All the tough ones have been covered under the given questions Keep in prayers feel free to discuss wish u a goodluck......

-=-=-=-=-=-=-=-=-=-=-=-=-=-=-=-=-=-=-=-=-=-=-=-=-=-=-=-=-=-=-=-=-=-=-=-=-=-=-=-=-=-=-=-=-=-=-

73

(6). Surgery 7th March 2016 (Morning) by By Haris Riaz Sheikh (156+)

1.Pt had anorexia diarrhea ,Alpha feto normal ,CEA raised,Bilurubin 16,GGt raised,Alt ast normal,Diagnosis ? a.-CA Liver b.-CA colon c.-Ulcerative colitis 2. Somatostatin dec insulin in which fashion ? a.Endocrine b.Neuroendocrine c.Apocrine d.Holocrine e.Paracrine 3. Old age pt presented wid chest pain from last 4-6 hrx at 2AM Emergency managemnt done Labs were Na 141 K 4.1 Sugar 103 Which will make diagnosis ? a.Cpk b.Ldh c.Ck mb 4. Prozone phenomenon- a. inc antigen titer-b. inc antibody titer- c. Dec antibody titer 5.90 years old lady has bruise on hand n arm...she has no other complaint A thrombocytopenia B treatment vik k C capilary defect 6.Pt had epidural anesthesia feeling tingling sensation in thumb Nerve involved C5 C6 C7 C8 7. Adaptation of receptor: A dec sensation bcx of change in concentration to sum other sensory input B den in sensation bcx sensory input is repeated over time Sum other options like that dnt remember exactly 8. Regarding breast -Tubuloalveolar glands -2nd to 6th ICS -15-20 lacteferous ducts

9. Extraembryonic coelomic membrane derived from Epiblast Hypoblast Endoderm

10. After splenectomy child is least susceptible to following infection A streptococcus B H influenza C pneumococcal D e coli E Neisseria

11. Ureter is posteriorly related to Ovarian vessels Genitofemoral nerve Root of mesentry Diaphragm 74

12. Regarding trachea Moves down during deglutition Bifurcates at T2 Fixed in children n mobile in adults 18cm in adults Enclosed in prevertebral fascia

13. Descending thoracic aorta A becomes abdominal aorta at L2 B Begins at upper border of body of T4 C Supplies lower 8 intercoastal spaces on both sides D Supplies esophagus pericardium n lungs E lies in middle mediastinum

14. Vagus nerve Supplies longitudinal muscles in esophagus Leaves the skull through middle of jugular foramen

15. Girl feels pain in one half of neck...pain is due study related...pain will relieve by dividing which nerve Phrenic Spinal accesory Vagus

16. Pt had nodule on ant 2/3 of tongue...lumphatic drainage to Superior deep cervical Inferior deep cervical Submandibular submental

17. In von willebrand disease A Inc pt B inc aptt C Inc BT D Thrombocytopenia E Normal platelets count

18. Congenital cataract cardiac defects n deafness related to A maternal rubella

19. Haesselbach triangle Femoral hernia Direct inguinal Indirect inguinal

20. Internal spermatic fascia derived from External oblique 75

Internal oblique Transversallis fascia

21. Left shift Hb dissociation curve A 2-3 biphosphoglycerate B altitude C inc temp D HbF E acidosis

22. Most obvious ecg changes in MI Tall P waves Inc PR interval ST elevation

23. Scanty barr body Turner Klinefelter syndrome

24. Male wid small legs n small arms, head n trunk normal in size , intellectual normal A autosomal dominant B autosomal recessive C X linked recessive

25. First sign of lidocaine toxicity A Light headedness B tonic clonic seizures C nausea vomiting

26. Exposure to hydrocarbons causes A seminomas B acute leukemias C bronchogenic carcinoma

27. Hypermagnecemia causes A hyperreflexia B dec acetylcholine C inc acetylcholine n muscle twitching

28. Superficail temporal nerve is açcompanied A auriculotemporal nerve

29. Second arch derivative A stylohyoid ligament B stylomandibular ligament C thyroid cartilage 76

30. Lymphatic nodule not present in A lymph node B thymus C spleen

31. Glucose as sole source of energy in Neuron Skeletal muscle

32. Following transplant most common tumour is Renal Hepatic Leukemias Skin malignancy

33. Which of the following inc GFR Bowmann capsule pressure Efferent constriction Dec in plasma proteins Constricting external urethra

34. Blood group antigens are A polysaccharides B glycolipids C polypeptides

35. Which will cause severe form of steatorrhea A gastric resection B zolljnger ellinson syndrome C gall bladder resection D pancreatic resection

36. Baby at 6 weeks of age previously healthy presented first tym wid jaundice n bilirubin in wet diaper A crigler najjar B gilbert C biliary atresia D physiologic jaundice

37. Imp in wound healing A collagen I B collagen ll C collagen lll D collagen lV E collagen V 77

38. Glycocalx is A role in absorption B a glycoprotein C has a positively charged sulphate group

39. Regarding motor neuron A single nerve supplies many muscle fibres B many nerve fibers supply single muscle fiber C single nerve fiber supplies many muscle fibers

40. During appendicectomy at mcburney point the injury to A deep circumflex artery B inferior epigastric artey C genitofemoral nerve

41. Rupture of bulbar urethra urine goes A superficial perineal pouch B deep perineal pounch

42. Levator ani supplied by A pudendal nerve B pudendal nerve via sacral 2 C Obturator nerve

43. Sigmoid colon supplied by A L3,4,5 B S1,2,3 C S2,3,4

44. Pain fibers travel thru A B fibers B A alpha C A delta D C fibers

45. Presynaptic cholinergic fibers travel thru A B fibers B A alpha C A delta D C fibers

46. Cholinnergic fibers causes A gall bladder relaxtion B detrusor relaxtion C ciliary muscle contraction 78

47. Which of the following causes septic sepsis EBV streptococci CMV

48. Old man presented wid persistant cough n thick mucoid sputum A mycobactrium Tuberculosis B H influenza C pseudomonas D klebsiella

49. Subacute endocarditis A spreptococcus viridians B staph aureus

50. Scenerio of endocarditis most common organism A staph aureus B viridian grp

51. Female taking contraceptive pills will have inc risk of A thromboembolism B osteoporosis

52. Female taking estrogen to relieve post menupausal symptoms will inc risk of A thromboembolism B cerical carcinoma C osteoporosis

53. Pt has dilatd umbilical veins will have bcx of which structure A hepatic veins B portal vein C hepatic artery D sup mesenteric artry

54. A patient has dependnt edema is diagnosed as having hepatoma caused by compression of A portal vein B IVC C hepatic artery

55. 1st cervical vertebrae has which of following A absent foramen transversarium B has odontoid process C lacks body D massive body

56. Basic drugs bind preferentially with A albumjn 79

B alpha globulin C beta globuljn

57. In our culture support is provided by A Volunteer B edhi centre C family mambers D health care professional

58. Confidentiality can b breached when A pt has treated priviously wid sum unknown drugs B pt is from sum other country C cannot afford D when he allows to do so

59. subArachnoid space ends at L2 L4,5 S1,2 S3,4

60. Clara cells are present in A terminal bronchioles B respiratory bronchioles C larynx D trachea E bronchi

61. Meckel diverticulim Vetilline ducts

62. I-CAM V-CAM leukocyte adhesion molecules

63. Cervical epithilium stratified squamous Metaplasia Anaplasia Dysplasia

64. Epithelium in lungs of a smoker A stratified squamous continuous B patchy stratified squamous with stratified columnar

65. Sec paristalsis will b observed when A achalasia esophagus B vagotomy C distension of esophagus 80

66. Columnar epithelium in esophagus Adenocarcinoma

67. Microscopic Diagnosis of tb A AFB B casseous necrosis

68. Hilar mases n non casseating granulomas Sarcoidosis

69. Dec absorption of bile salts by resection of A ilium B jejunjm C colon

70. Pleural effusion Upper border of lower rib at the point of max dullness

71. Aneurysm descending thoracic aorta will compress Azygous vein n thoracic duct

72. Aortic aneurysm Medial necrosis

73. Cardiac reserve A amount of blood that can b pump after normal flow B cardiac out per total body surface area

74. Anaphylactic shock differs from hypovolumic shock by A inc carfiac output B inc peripheral resistance C inc heart rate

75. A boy has blood loss n doctor do not find any vein for iv maintainance...ideal vein for venesection Great saphenous

76. A female has post partum hemorrhage n diagnosed has having acute rrnal failure...the structure most probably affected is A PCT B DCT C collecting ducts D asc limb of loop of henle

77. Which of the following is different from plasma osmotically A 0.9% N/S B 5% D/w 81

C Plasma protein fraction D Ranger lactate

78. Vasopressin n oxytocin originate in A Hypothalamus B ant pituitary C post pituitary

79. Vasopressin acts via A osmoregulation B sympathetiv regulation C pH

80. Cerebral blood flow is maintained by A sympathetic discharge B pO2 C pCO2

81. Dec in functional hemoglobin as in anemia Carbon monoxide poisoning n methemoglobenemia will not causes hyperoxia bcx A stimulated by central receptors B stimultd by carotid body C arterial pO2 remains normal D arterial oxygen content is kept in normal range

82. Serum iron stores as Ferritin Transfirrin Iron

83. Inc need of iron during pregnany is represnted by inc in A iron B feritin C TIBG

84. Myxedema is caused by A total lack of thyroid hormone B partial lack of thyroid hormone C lack of hypothalamic pituitary D aldosterone E ADH

85. Fisherman wid sensory loss Diphylobothrium latum

86. Scenerio showing necrosis of brain Liquifactive necrosis 82

87. Superficial temporal artey accompanies Auriculotemporal nerve

88. Left ovarian artery is a branch of Abdominal aota

89. Most common lesion caused by chewing bettle nuts pan submucosal fibrosis

90. When blood is acidified A dec bicarb excretion B inc H reabsorbtion C aldosterone

91. Fracture neck of femur most common complication AVASCULAR NECROSIS

92. Pseudomembranous colitis is associated wid antibiotic use A clindamycin B cefulexin C gentamycin D erythroxycin

93. ADPKD associated wid renal failure Dnt remember other options

94. Calculate median in following data... Total 8 values middle two were 25 n 30 So median was 27.5

95. During ka study of two populations in which population having risk factors in history developing CHD n other population who do not have risk factor study is called (sumthing like that) A case control B cohort C random ampling

96. Accessory spinal nerve to sternoceidomastoid accessory is motor supply,,,,,,C2 ,C3,,, right answers but they both were given as separate options

97. In spermiogenesis acrosomal cap is formed by A lysosomes B mitochondria C golgi complec D Endoplasmic reticulum 83

98. A pt was prescribed tricyclic antidepressant ...its analgesic action is mediated in A 1 day B 3,4 days C 3,4 weeks D more than 6 weeks

99. There was question of cerebellum that do appropriate action of motor cortex. .. i marled climbing n mossy fibers...statement ni hae mind mei

100. Penetrating injrury to 4th incoastal space jst on left side of sternum will damage A right atrium B intercoastal musvle C intercoastal membrane

101. Posterior rectus sheath end as--Arcuate line

102. Most common breast tumour A carcinoma in situ B invasive ductal carcinoma C comedocarcinoma D lobular carcinoma E indraductal pappiloma

103. In polycythemia vera which cells not raised A neutrophills B lumphocytes C monocytes D eosinophils E basophills

104. Posterior relation of hip joint A sacral nerves B piriform bursa

105. Rupture of posterior cruciate ligament will result in A ant displacemnt of tibia B ant displacemnt of femur

106. Pt of hurshsprunge disease of sigmoid colon which spinal nerve supply . S1.s2.s3 S2.s3.s4

107. Coibital fossa nidal pass which nerve demage. . Median nerve

108. Myelin sheath is formed by A oligodendrocytes 84

B schwann cells C astrocytes

109. Exocelomic membrane derived from ? I marked a.hypoblast b.meso?

110. Middle meningeal artery Foramen spinosum

111. High canc of co2 in which .. Pulmonary vein Pulmonary arteries Experied air..

112. Which organ can not regenerate ? A lens B neuron C skeletal muscle

113. Regarding symphysis pubis

Synarthrosis Gomphosis Sec ossification

114. Q abt abducting of shoulder intial 30 degree. .. Supraspinatus Infraspinatus Delatiod

115. Q.abt injury to hip joint capsule .. Ans .avascular necrosis

116. A patient is askd to lower his arm slowly from right angle but when he does so drops his arm Deltoid

117. Which of the following has shortest pre erythrocytic phase A falciparum B vivax C malariae D ovale

118. Post natal conjuctivitis A Gonorrhea B chlymadia trachomatis C HSV 85

119. Pt having multiple polyps in colon ...his grandfather died of similar illness Adenomatous polyps

120. Which drug cause psodo membranous colitis. . Vencomycine Clandamycine

121. Diagnostic of thalasemia major A Hbf more 90% B iron deficiency anemia

122. Pt had anorexia diarrhea Alpha feto normal CEA raised Bilurubin 16 GGt raised Alt ast normal Diagnosis ? -CA Liver -CA colon -Ulcerative colitis

Ulcerative colitis due to involvement of biliary system Primary biliary cirrhosis Thats y bilirubin is raised n also Ggt

123. Breast tumour (T3,N2,M0) Stage ? A stage ll a B stage ll b C Stage lll D stage IV E stage l

124. During thyroidectomy superior thyroid artery is ligated close to gland to save A external laryngeal nerve B Recurrent laryngeal nerve C internal laryngeal nerve

125. Large amount of dextrose infusion given inhibit which hormone ? ADH

126. Phopholipase A2 is inhibited by -Corticosteroids

127. Clostridium botulinum causes Flaccid paralysis

128. Something regarding atropine ?

ANS was it counters the effectsof parasympathetic nervous system in effector orgens 86

129. Medial meniscus attachd wid -Medial colletaral ligament

130. Structure which remains in true pelvis-Middle rectal artery ?

131. Lymphatic drainage of cervix A internal iliac B external iliac C deep inguinal D superficial inguinal

132. Pt had abdominal surgery for mass for which he remained hospitalized ...he got some cardiac abnormality...his labs shows inc potassium Most probable cause of this inc is A prolong antibiotic use B postoperative infection C prolonged bedrest

133. Antibiotic should b given A 6 hrx before surgery B 12 hrx before srgery C on operating table D after the surgery E on first dressing

134. Substance which is maximally reabsorbed by the kidney A Na B K C glucose D urea E bicarb

135. Most common factor for a tumor to b called malignant Metastasis

136. Pain is reffered to shoulder by A C2,3,4, B C3,4,5

137. Hydrocephalus most commonly caused by stenosis of A duct of sylvier B formen lushka C foramen mejendie

138. Very small particles in respiratory tract are cleared by A cilia B mass transport C phagocytosis 87

139. Surgeon accidently removed parathyroid gland during thyroidectomy Hypocalcemia numbness n positive chivostek sign

140. All the short muscles of hand are supplied by A median nerve B ulnar nerve C radial nerve D T1 E C5

141. Perforation of duodenum most common source of bleeding-Gastroduodenal artery

142. Hereditary spherocytosis is defect of--Structural proteins

143. Regarding RBC fragility-0.35% N/S

144. H202 present in-Peroxisomes

145. Regarding irreversible injury of cardiac cell A Cell swelling B clumping of chromatin material

146. 40 years old female having 4 children has cervical epithelial change--HPV

147. Golgi tendons- Tension in muscles

148. Most common cause of hyperthyroidism--Graves disease

149. Exogenous steroids will dec which one of following--Corticotropin

150. Q there was 2 q abt autosomal dominat disease --Acgondroplasia -Adult polycystic disease

151.Q was abt renal faliar hemorrhage which area of nephron effected a.PCT,b.Collecting duct ,c.Asanding loop henle

152.old man cough with thick mucoid sputum?A ) Klebsiella, B) mycobacterium,3 pseudomonas

153. EARLIEST SIGN OF LIDOCAINE TOXICITY???,,,,A.) Lightheadedness,,,,B) Tonic clonic seizures

154. Q: A man develops pain in calf muscles after walking for 100 yards , which settles after taking rest.. which artery involved? Anterior perforating, popliteal , int iliac, ext iliac, common iliac ??

155. Q was abt which type of collagen fiber take part in wound repair Tpye 1.2.3.4 ,I mark type 1

156. Tissue taken for transplantation from identical twin...true statement A never rejected 88

B require immunosuppesants C always rejctd due to minor HLA incompatibility

7th March 2016 (Morning) Surgery by Umeed-eSubah

1.Prozone Phenomenon 2.In Our Culture Patient Support (Edhi, Social Voulteer, Family) 3.Basic Drug Binds (Albumin, globulin, alpha glycoprotein) 4.Portal Hypertension causing dilated Para umblical vein due to (Portal Vein, Hepatic Vein, Inferior Vena Cava, Hepatic Artery) 5.Bulbous Urethra Rupture - Urine Extravasate into - (Deep Perineal Pouch, Superficial Pouch, Ischiorectal fossa, urogenital diaphragm) 6. Baby Diper gets yellow due to (Gilbert, Crigller Najar, Biliary Atresia, Hemolytic disorder) 7.Fast Pain Fiber (A alpha,Beta,delta, B or C) 8.PreGanglionic Nerve Fiber (A alpha,Beta,delta, B or C) 9.Cervical Carcinoma - Lymph drains (Internal Iliac, External Iliac, deep inguinal, superficial inguinal, paraaortic) 10. Pleural Drainage (Area of Max Dullness upper border of lower rib, Midclavicular line 2nd intercostal upper border, lower border) 11. Azygous Vein and Thoracic Duct Accompanies Aorta 12. Aortic Aneurysm (Medial Necrosis) 13. Left Lung notch formed by (Acending , descending or arch of aorta) 14. Arachnoid space ends (L3-L4, L5, S1-S2, L2) 15. Pudendal nerve in Pudendal canal in ischiorectal fossa 16. Which does not leaves pelvis (obturator nerve, pudendal nerve, ilioinguinal nerve,inferior gluteal ) 17. Old age with gastric discomfort - bruise over hand - Treatment Vit K 18. short arms and legs - long trunk - (X Linked, dominant, recessive, Inherited) 19.Bilesalts absorb from (Ileum, jejunum, duodenum, colon) 20.CEA+, alpha feto protein, ALP Raised, ALT Normal, AST Normal ( CA Colon, Hepatoma) 21.Family history positive , 100 of polyps (Adenomatous, Hamartomatous,) 22.Esophagus Epithelium changes to columnar epithelium- Leads to (Adenocarcinoma, Squamous cell carcinoma) 23. Cervical epithelium Stratified sqaumous ( Dysplasia, Metaplasia) 24.Tuberculosis Diagnosis (AFB+, Caseous granuloma) 25.autonomic supply from (L1,L2,L3 - S1S2S3 - L2L3L4) 26.Meckel Diverticulum (Allantois, Vitelline) 27.ICAM and VCAM- causes Leukocyte adhesion 28. Clara Cells presents in (Terminal Bronchiole, Respiratory Bronchiole, Bronchi

1. Council man bodies are found in ? 2. Barr Bodies in Klinefelte 3.Left Shift due to (Acidosis, FHb, Inc 23BPG, Inc Altitude Inc Temp) 4. hassel Bach Triangle ( Lumbar hernia, Femoral, Direct Indirect 89

=-=--=-=-=-=-=-=-=-=-=-=-=-=-=-=-=-=-=-=-=-=-=-=-=-=-=-=-=-=-=-=-=-=-=-=-=-=-=-=-=-=-=-=-=-=- (7). Gyane/Obs 7th March Morning by Noor Fatima 1- 1 gram protiens give ? 4kcal 2- Regarding thyroid? Drain in deep cervical lymph node 3- Subarachnoid space ends at ? S2 4- Medial quadrant of breast drain into? Internal thoracic nodes 5- Lateral quadrant of breast drainage?anterior pectoral nodes 6- Symphysis pubis ? secondary cartilaginous joint 7- In males rectum separated from bladder by? Fascia of denonvilliers 8- 1/3rd of total body water?ecf 9- Muscle of sadness and fright which lower angle of mouth slightly downward? Platysma 10- Sacral plexus anterior division branch?a)femoral b)obturator c)post cutaneous n of thigh d)superior gluteal e)inf gluteal 11- Portal hypertension ,vein dilated?a)inf epigastric b)inf mesenteric c)esophageal 12- Superficial temporal artry accompanied by? Auriculotemporal nerve 13- Common motor pathway?a)descending corticospinal tract b)motor cerebral cortex 14- Intratesticular temperature less than abdominal cavity by?a)1-2 b)2-3 15- Glycocalyx is? Carbohydrate moiety b)charged with sulphate group 16- Hypermagnesemia causes? Dec acetylcholine secretion(bcz it causes dec calcium,dec presynaptic calcium entry nd so decreased ach) 17- Ureter most narrow at?a) uretropelvic b)when enter in bladder c)at pelvic brim 18- Brain primarily regulated by?A)pH B)Pco2 19- OBJECTives of health programmes are mainly? A)Qualitative statement b)quantitative c)not influenced by wht hv achieved previously 20- Confidentiality breaks?if pt authorizes u( insurance claim was nt in option) 21- Post streptococcal glomerulonephritis?subepithelial deposits 22- Nitrous oxide safe level in operation theater per day? A)100ppm b)10ppm c)200ppm 23- Inguinal canal posterior wall formed by?cojoint tendon 24- Maternal mortality per?a) 100000 b)1000 (no option of 10,000) 25- Epidural anesthesia given,to inc its duration nd to achieve more sensory nd motor blockage?a)inc dose b)inc conc c)give with 1;20000 epinephrine 26- 40 year male with yellow sputum b/l lung infiltrates nd left upper lobe consolidation with air fluid level,in sputum neutrophils present?a)mycoplasma b)staph aureus c)tb d)adenovirus 27- Motor unit? Muscle fascicle supplied by one motor nerve 28- There is no syphatetic drive nd circulatory collapse occur d/t? a)loss of vasomotor tone b)loss of venous tone 29- Mifepristone ?antiprogestin 30- Tamoxifen ?estrogen agonist 31- Osteoblasts are multinucleated ? or uncalcified organic matrix Is called osteoid? 32- Stylohyoid ?second arch derivative 33- Pre-eclampsia triggering factr is?endothelial alteration b)fetoplacental circulation compromised 90

34- Bite cells? G6pd def 35- Blood group antigens on rbcs are?a)glycolipid b)glycoprotein (no option of glycosphingolipid) 36- Brain autopsy done?liquefactive necrosis 37- For transfer of radicals(atoms)from one group to another?a)oxidoreductase b)transferase 38- For conversion of glucose 1 phosphate to glucose 6 phosphate require? A)atp b)isomerase 39- Acute beta 2 blockage causes?a)hypoglycemia b)lipolysis )vasocons in muscles 40- Common cause of non obstetrical death in our country? A)domestic violence b)heart disease c)tb d)asthma 41- Most common cause of maternal mortality in our country? A)sepsis b)haemorrhage 42- Eating raweggs causes?tapeworm infestation 43- Calculate median (total values were 8,nd middle two were 27,30..median will 27.5) 44- In malaria cause of anemia? A)autoimmune b)hemolysis of para sitized rbcs 45- Golgi tendon organs?muscle tension 46- Lymph node biopsy shows non caseating granuloma?a)sarcoidosis b)amebiasis c)histoplasmmosis 47- Most frequent is?a)autosomal dominant b)aut recessive c)x-linked d)chromosomal e)multifactorial 48- Warfarin overdose causes brain haemorrhage immediately give?a)vit k + ffp b) ffp 49- Superior sagittal sinus pressure in standing position?a)+10 b)- 10 50- After ATN which part affected most?a0pct b)loop of henle c)dct 51- Deep ring in fascia transversalis 52- Aspiration of hydrocele,needle does nt pierce?tunica albugenia 53- Councilman bodies?apoptosis 54- Preterm labor? Bacterial vaginosis 55- Breastfed recommened ?within half an hour after delivery 56- Dvt embolus goes into?pulmonary artry 57- pCO2 max in?a)expired air b)insp air c)pulmonary artry d)pulm vein 58- dec release of o2 from hemoglobin d/t? a)dec temp b)inc temp c)hypoxia 59- pain on flexing leg medialy ?pelvic appendix 60- shoulder pain reffered from pleura?c3,c4,c5 61- nerve supply of fibrous nd parietal pericardium?phrenic 62- ureter is in front of?common iliac artry bifurcation 63- ureter is?a)away from utrus b)superior to uterine vessels 64- left suprarenal drain in?left renal vein 65- lymphocyte production nd turnover regulated by?a)thyroid b)zona fasciculate 66- most toxic local anesthesia?bupivacaine 67- vagus nerve?passes through middle of jugular foramen b)beneath subclavian artry 68- burkit lymphoma in AIDS? EBV 69- screening test ?acceptable to general population 70- screening done to detect disease at early stage 71- pain from fingertips by? Abeta fibrs 91

72- NTD? Alpha fetoprotein raised 73- Aftr c section pain in lumbr region?ureter ligated 74- After ejaculation sperm may b stored into? A)ampulla of ejaculatory duct b)seminal vesicle 75- Cervix lymph drain?internal iliac nodes 76- Boy with testicle swelling,fever ,b/l mandibular angle swelling?mumps 77- Hemophilia gene located on? X chromosome 78- Middle meningeal artrt passes through ?foramen spinosum 79- Labia majora swelling lymph passes to?superficial inguinal medial group 80- IVC commences at?L5 81- Testicular lymph drain into?paraortic nodes 82- Muscle damage in obstructed labor?levator ani 83- Symp supply of midgut? 84- Newborn baby with absent limbs?amniotic band syndrome 85- Thyroid hormone causes?inc FFA 86- Which causes dec production of prostagalndins? A)asprin b) steroids 87- Metaplasia? Functional change in epithelium 88- Vagina? St sq epi 89- Most common cause of male infertility?klinfilter synd 90- Semen analysis abnormal finding? 91- Newborn baby with abdominal swelling covered with membrane?omphalocele 92- Mensturation at 18 day?secretry phase 93- After birth ossification centr present already at?lower end of femur 94- Ist sign of vit A deficiency? Night blindness 95- Pt stand on one foot,drop on left?ritegluteus medius paralysis 96- Fallopian tube?columner epi 97- Goblet ells absent,clara cels present?a)terminl bronchiole b)trachea 98- Bulbourehral gland content of deep perineal pouch 99- Adh nd oxytocin production in?hypothalamus 100- Jaundice in pregnancy?GGT 101- In pregnancy obliteraton of utrus by?decidua basalis capsularis b) capsularis nd parietalis 102- Memory ?hippocampus 103- Hypothyroidism drug of choice?levothyroxine 104- In diabetic postprandial?inc insulin nd inc glucose 105- Virus causes protein alteration 106- Radiation cause hange in prorto-oncogene 107- Gastric carcinoma ?h pylori 108- Inc BT ?thrombocytopenia 109- Basic drugs bind with?alpha glycoprotein 110- ICAM nd VCAM ?adhesion molecules 111- Pyogenic peritonitis>?bacteroids 112- Diff b/w benign nd malignant tumor? Metastasis 113- Microscopic fature of malignant?invasion 92

114- No lymphatic nodules in?thymus 115- Short stature? Autosomal dominant 116- Pt lying naked heat loss by?conduction 117- Congenital cataract ?rubella 118- Above hilum of left lung ?arch of aorta 119- Pt supported in our country by? A)Edhi b)family 120- Fast pain ?a delta 121- Preganglionic fibers are? B type 122- Pleural drainage site?upper border of lower rib at max dullness 123- Tb confirmation? Acid fast bacillus 124- Somatostatin act on beta cells by?a)paracrine action b)endocrine c)neurocrine 125- Ventricular filling /s3 126- Acrosome reaction by?a)golgi b)lysosomes 127- Brain uses only fuel?glucose 128- Chest pain 6 hours ago? CK-MB 129- Cns myelination?oligodedrocytes 130- LMNL ?flaccid paralysis 131- Recptr acting via phospholipase? Vasopressin mediated sk muscle contraction 132- Neck of fibula fracture? Common peroneal nerve injured 133- In standing person venous return facilitated by? Sk mucles contraction in legs 134- Fright or flight?a) Dec release of bronchial secretions b)inc git motility 135- Atrial fibrillation?pulses deficit 136- Hot sunny day person collapses? Heat exhaustion due to excessive sweating 137- C1 vetebra have no body 138- Inc gfr? Dec plasma proteins 139- Cardiac reserve? 140- Osmosis depend on ?osmoreceptors 141- Betelnut chewing?submucosal fibrosis 142- Tb antibodies are? Cell bound 143- In HIV ? dec helper cells 144- In iron deficiency wht is increased? TIBC 145- Very small particles in respiratrytract removed by?a)ciliary movement b)phagocytosis 146- ADPKD associated with? A)cerebral haemorrhage b) renal failure 147- Nucleus containing organelle?mitochondria 148- Pseudohermaphrodite?46xxy 149- Aorta give renal artry at?L 2 150- Infection of pretracheal fascia extend to?anterior mediastinum 151- Systemic fungal infection drug of choice ?amphotericn 152- Gastric ulcer pain? Greater splanchnic nerve 153- Withdrawl nociceptive reflex/? Multisynaptic 154- Opsonin ?c3b 155- Alpha adrenergic causes?mydriasis 156- T cells bind with ?MHC 93

157- Thyroid isthumus in front of?2,3,4 tracheal rings 158- Lt circumflex involved area affected? Lt atrium nd lt ventricle 159- DKA pt died d/t? mucormycosis 160- Hypertensive pt ,severe chest pain radiating to back ,died.on autopsy? Medial calcific necrosis 161- Abdominal angina d/t? superior mesenteric artry 162- In pre mature baby common is? PDA 163- Lumber puncture?ligamentum flavum punctured 164- Testosterone ?inc bone nd muscle growth 165- Oral anticoagulation monitored by?PT 166- Dvt d/t use of? OCPs 167- Liver transplant? CMV 168- Cholangiocarcinoma? Chlonorchis sinensis 169- Femoral artry at?midinguinal point 170- Dec ESR d/t?inc albumin 171- Max clearance of?PAH 172- Rt testicular drain into ?IVC 173- Organelle with double membrane ?nucleus 174- Post communicating?connects PCA with ICA 175- Uncoupling of phosphorylation with inc heat production by?thyroid hormones 176- P53?tumor suppressor gene 177- Most radiosensitive?seminoma 178- Corynebacterium diphtheria most lethal effect on? Heart 179- Sulfer containing amino acid? Cysteine 180- Prim active transport require?pump 181- Fisherman bleeding gums? Scurvy 182- 182-hematocrit 183- Edema due to lymphatic blockage 184- Typhoid in first week? Blood culture 185- Sodium regulation by?osmoreceptors

- = - = - = - = - = - = - = - = - = - = - = - = - = - = - = - = - = - = - = - = - = - = - = - = - = - = - = -

94

(8) Gynae / Obs- 8th March 2016; (Morning Session) Nourin Hameed (105) Paper A upto 70% and paper B upto 50% repeat tha. 1)Gram negative bacteria causes infection of which system A)Urinary tract B)Biliar tract 2)What is tachyphylaxis A)Diminished response developing rapidy B)Diminished response developing slowly 3. 3)Which vitamin is antioxidant A)Vitamin A B)Vitamin E C)Vitamin k 4)Vein accompanying anterior inter-ventricular vein A)Great Cardiac Artery B)Middle Cardiac vein C)Superior cardiac vein 5)Vessel accompained by Phrenic Nerve A)Musculophrenic Nerve B)Pericariophrenic artery C)Internal carotid D)Superior Epigastric artery E)Internal thoracic 6)Post ganglionic sympathetic fibres are present in A)All spinal level B)All thoracic level C)All cervical level 7)Patient came with vaginal dischrage,whitish curd like,causative agent A)Chlamydia B)Trichomoniasis C)Candida 8)Structure in the oral cavity covered by stratified squmous epithelium A)Palatine Tonsil B)Thymus 9)Postganglioic sympathetic nerves doesn't release A)Acetylcholine B)aDRENALINE C)Dopamine 10)Sulphur containg amino acids include A)Methionine B)Glutamate C)Lysine 11)Cause of Edema A)Decresed Hydroststic pressure B)Decreased Colloid pressure of ECF C)Lymphatic obstruction 12) Per mint perfusion of 100g brain tissue per minute A)10-15 ml B)20-30 ml C)50-55ml 95

D)95-100 ml E)100-105 ml 13)Glucocorticoids a. Decrease FA uptake by liver b. Decreasd lipolysis c. Deceased proteolysis d. Decrease glucose utilization by cells 14) Myxoid degeneration is mostly associated with a) carcinoid valve disease b) infection endocarditis c) libman endocarditis d) marantic endocarditis e) mitral valve prolapsed 15)Common Longest bone fracture Tibia Femur Clavical Humerus 16)-Howship lacunae contain Chondrocyte Chondrblast Osteoblast Osteocyte 17)Internal carotid different from extrrnal carotid by A)Branches in neck B)More pulsatile C)More deep in the neck 18).Disease in which immunological test is of diagnostic value A.hydatid disease B.amoebic liver abcess 19)First bone ossified is A)Clavicle B)Humerus C)Sternum 20)Femoral Nerve root valve A)L3,4,5 b)s1,s2 C)L2,3,4 21)Tunning fork placed on head sensations percieved by Paccinian corp Merkle Messiner 22)abt trigeminal nerve Divide in merkl cave Hv 3 nuclei Gives sensory brnch to angle of jaw 23)Last end of spinal cord surrounded by dura,pia,arachnoid end at the level of A)S1 b)s2 C)L1 96

D)L2 E)Coccygeal level 24)Vein accompanying anterior interventricular artery A)Middle Cardiac vein B)Great cardiac vein C)Superior Cardiac vein 25) Right border of heart on chest X-ray is partly is formed by a- right ventricle b- SVC c- SVC+Rt Atrium 26)True regarding thyroid gland A)Epitheilum changes its shape B)Isthmus lies above cricoid cartilage 27)Breast medial/Lower compartment doesnot A)Pectoral lymph nodes B)Supraclavicular lymph nodes C)Inferior phrenic lymph nodes 28)Structure present in the lateral vaginal fornex A)Ureter B)Fallopian tube C)Ovaries 29)Paccinian corpuscles respond to A)Rapidly adapting vibration B)Slowly adapting vibration C)Touch D)Two point discrimination 30)Free nerve endings A)Detect only pain B)Are non encapsulated 31)Pus contain A)Dead bacteria B)Dead neutrophils 32)Structures not present in post ganglionic sympathetic fibres A)Noe-adrenaline B)Adrenaline C)Dopamine D)Acetyl Choline 33)True about CSF A)Inc K than Plasma B)Decreased Glucose C)Decreased Specific gravity D)Decreased chloride than plasma 33)Which part of brain has role in memory A)Frontal lobe B)Parietal lobe 34)RBCS are A bioconvex B nonfragile C have nucleus D have glycolytic enzyme activity 97

35)erythropoietin is sec by mesengial cells peritubular 36)Factor viii is produced by: a. hepatocytes b. kuffer cell c. bone marrow d. spleen e. Thymus 37)Patent with low Hb and had mulpitiple ulcers the body ,type of anemia A)Sickle Cell G6Pd deficiency anemia 38)Patient with jaudice.Had low Hb and increaed reticulocytes count.Type of anemia A)Hemolytic anemia B)G6PD deficiency anemia 39)Muscle hepling in opening the jaw A)Medial Pterygoid B)Lateral pterygoid C)Cricothyroid 40)True about paltelets A)4-8 micron in size B)alpha granules release serotinin C)Half life is about 10 days D)Count increases after splenectomy 41)Tubular structure having tubular glands in laina adventitia and surrounded by three layers of smooth musvles A)Ureter B)Urinary bladder 42)Growth hormone secretion is inhibited by A)Sleep B)Somatostain C)Pubert D)Exercise 43)Hormone causing incresed follicular growth and maturation A)Lh B)FSH C)Estrogen D)HCG 44)Pregnat lady had viral infection during pregnancy.She was worried when she read on internet taht a type of viral infection is lethal in pregnancy.Which type of viral Infection could that be? A)Hep A B)Hep B C)Hep D D)Hep E 45)Patient had HSV infection.Epithelium changes its shape with increased tendency of metastasis.Type of Change? A)Dysplasi B)Metaplasia C)Hyperplasia 98

46)Myeloperoxidase is present in A)Platelets B)Basophils C)Neutrophils 47)Type of hypersensitivity in erythroblastosis fetalis A)Type 1 B)Type 2 C)Type 3 D)Type4 48)Patient had sore throat and streptococcal infection.He may also be suffering from A)Hepatitis B)Gastroenteritis C)Glomerulonephritis 49)Which of the fallowing is carcinogen A)CO B)Asbestos C)cisplatin 50)Bladder ca is caused by A)Schistosoma mansonin B)Schistosoma Hematobium 51)Allele is A)Non fragmented gene B)Non identical gene C)Structural gene 52)Characteristic of malignancy A)Pleomorphism B)Invasion 53)Hamartoma is A)Malignant tumor B)Totally benign 54)Protein concentration in exudate A)1-2 gm B)2.5-4 gm C)Above 4 gm 55)Patient had high Calcium,low phosphate,high parathomone,Cause? A)Primary Hypoparathyroidis B)Secondy Hypoparathyroidism C)Chronic Renal Failure D)Vitamin D deficiency 56)Patient is on Glucocorticoid for long time.She will have all of the fallowing Except A)Moon facce B)Hpotension C)Thin skin D)Hypertension 57)Most important thing in doctor patient relation is A)Mutual relation B)To clarely explain the disease situation 58..which of the following brain vesicle first appear A.telencephalon B..diencephalon C.metencephalon 99

D.myelencephalon E.mesencephalon 59.parathyroid related question A.supplied by superior thyroid artery B. Present posterior to thyroid lobes 60.aortic aneurysm at aortic opening in diaphram Which structure will b compressed A.azygous vein B.hemiazygous C.azygous+thoracic duct 61..incidence ki definition was in option 62.most common prim cause of HCC in developing countries A.hep B &C B.hep B C.Hep C D.alpha 1 antitrypsin deficiency 63.a boy with fracture,serum calcium was 8.2 Excrete calcium in urine A.vit D deficiency B.prim hypoparathyroidism .Bmr in resting condition maintained by A.liver B.brain C.heart 65.intervertebral disc forms 1/4 of vertebral columb 66..loss of nuclei in RBCs occur at which stage A.reticulocyte B.proerythroblast 67..septic shock mostly due to A.gram negative B.gram positive 68.More than 90% patient of anthrax having manifestation of A.skin 69..stimus fr central chemoreceptos A.pco2 70.1 litr of normal saline infusion causes A.inc plasma osmolarity B.dec urine osmolarity C.inc urine osmolarity 71.pulm embolism cause A.thromboembolism 72.Lobar pneumonia inveatigation was asked A.sputum culture B.blood culture 73.cell swelling sec to hypoxic injury cause will A.entery of water in cell B.protein accumulation in cells C.lipofuscun accumulation D.glycogen break down 74.in black woumd healing was asked, healng will A.keloid 100

B.premalignant lesio C.premalignant change 75..thorn prick abcess A.staph aureus 76.cell junctions dont allow movement A.gap junction B.tight junction 77..in human blood A.wbcs more than rbc B.platelets more than rbc C.iron binds to hemoglobin 78.a terminal ill patient with t3N1M1 stage, having less than five yr survival, What wll b seen commonly A.cachexia B.Hypertension 79.Not synthesized by postganglionic sympathetic A.dopamine B.adrenaline C.ac 80..increased bioavailibility A.metoclopramide B.Cyclizine C.Dapsone 81..medical ethics A.same as hippocratic oath B.code of conduct of doctor

82.True about anal canal A)Upper part drained by superficial inguinal lymph nodes B)Supplied by both superior and inferior Rectal arteries C)Inferior Rectal vein drains into Infeior mesenteric vein 83.True about Second degree heart block A)Shortening of PR interval B)Ventricular depolarization pattern is disturbed C)More prone to develop ventricular tachycardia 84.True about Progesteron A)Plays no role in breast development B)Not secreted in pregnancy C)Increased amount results in onset of menstruation D)Decreased amount results in onset of menstruation E)Increased amount causes abortion 85.Young patient with peritonsillar abscess.Causative agent? A)Diphtheria B)Streptococcus C)Staphlococcus 86.Aspirin and other NSAIDS work by acting on A)Lipoxygenase B)Cyclo-oxygenase C)HMG Co A reuctase inhibitor 87.If ovarian cycle duration is 21-23 days ,when will the ovulation takes place A)7-9 days 101

B)14th day C)18-20th day D)19-21 th day 88.Substance not increaed during pregnancy A)Thyroid hormone B)Growth hormone C)TIBC 89.Drug used to treat Endometriosis A)Danazole B)Flutamide C)Tamoxifen D)Ketoconazole 90.Patient presented with butterfly rash and proteinuria.The most diagnostic investigation would be A)Anti-double stranded DNA b)ASO titre C)LE cells 91.Toxoplasmosis is A)A parasite B)A helminth C)A virus D)A bacteria 92.Trachoma is caused by A)Candida b)Chylymydia C)Streptococcus 93.Sympathetic adrenergic stimulation causes A)Contraction of radial musles of iris B)Glycolysis 94.Increase Ejection fraction causes decrease in A)End diastlic volume B)End systolic volume 95.Osteocytes convert into bone cells due to the effect of which hormone A)FSH B)Thyroxine C)Parathormone 96.Following has lymphoid tissue with surrounding stratified squamous epithelium ... A.. Lymph node B..palatine tonsil C..spleen D..ovary.. 97.Which brain vesicle appears first A)Telencephalon B)Diencephalon C)Metencephalon D)Mesencephalon 98.Iron stored in parenchymal tissue in the form of A)Hemosiderin B)Ferritin 99.Which ligament has both intra as well as extrapelvic extension A)Round ligament of uterus 102

B)Round ligament of ovary C)Suspensory ligament 100.Microtubules are present in A)Mitochondria B)Centrioles C)RER 101.Skin of scrotum is supplied by A)Genitofemoral nerve B)Ilioinguinal nerve C)Femoral Nerve 102.A woman in labor room had excessive bleeding and was hypotensive.She had IUD on ultrasound.Initial management would be A)Evacuation B)Fluid replacement C)Oxygen therapy 103)the ascent of horse shoe shaped kidney is prevented by a external iliac b inferior mesenteric 104)urogenital diaphragm attached laterally to? A.inferior remus of pubis and ischial ramus B.ischial spines C.ischial tuberosities D.obturator internus facia E.tip of coccyx 105)Correct sequence of events ?

A. Damaged valve, thrombus , perforation , emboli

B. Damaged valve , perforation , thrombus , bacteremia

C. Damaged valve , bacteremia , thrombus , perforation

D. Bacteremia , thrombus , perforation , emboli

= - = - = - = - = - = - = - = - = - = - = - = - = - = - = - = - = - = - = - = - = - = - = - = - = - = - = -

103

9. Radiology 7th March 2016 by loa loa and Asfandyar Bhittani (122)

1.Trigeminal ganglion location 2.which lies posterior to the posterior part of internal capsule- Ans.Optic radiation?? 3.testicular carcinoma 1st metastases Superfacial inguinal lymph nodes(ans)ref:robbins Renal lymph nodes Deep inguinal lymph nodes (paraortic was not in option) 4.anterior relation of left kidney attached directly without membrane Spleen Duodenum Pancreas(ans) Stomach 5.axons of vestibular nucleus ends at 6.lymph nodes of lower internal mammary lymph nodes into except Pectoral nodes (ans) Inferior phrenic nodes Supraclavicular noded Suprasternal nodes 7.pectrol lymph nodes drain maximum Major part of breast Upper part of anterior trunk(ans) Upper part of posterior trunk 8. About vagus nerve Leaves the skull through jugular foramen (ans) Relaxtion of esophageal muscles 9.fragile x syndrome Trinucleatide repeat 10.which structure passes anterior to the left lowest part of the pancreas head Sma(ans) Ima 11.which stucture passes through esophageal hiatus Vagus trunk right vagus(ans) Phrenic nerve 12.9 yr boy with joint and bone pain also anemic Sickle cell(ans) Beta thalessemia 13. L1 L2 ends at Inferior hypogastric plexus(ans) Inferior mesenteric plexus Superior mesenteric plexus 14.detrusor nerve supply Parasympathetic pelvic nerves (ans) Inferior hypogastric plexus 15.portal vein tributary Hepatic vein Inferior epigastric vein (ans) 16. Hepatoma will early compress Portal vein(ans) Cystic duct Hepatic artery 104

17.portal vein tributary Left colic(ans) Inferior epigastric vein 18.adrenocortical deficiency causes Hyponatremia (ans) Alkalosis 19.herpes zoster nerve involved Trigeminal nerve(ans) Facial nerve 20. Regarding inguinal canal Roof formed by conjoint tendon(ans) Extends from anterior superior iliac spine to pubic tubercle 21.which bronchas of lung is supplied by endartrial artey? Middle lobe Superior lobe Inferior lobe 22.after chemotherapy Apoptosis 23.cerebral blood flow regulation Pco2(ans) H+ Po2 24.malarial parasite transfer in which form in man Schizont Sporozite(ans) 25.in embyo lung is covered by mesotheliam which later form which layer Visceral pleura( i marked)??? Parietal pleura Costodiaphragmatic recess 26.anterior to pancreas is Stomach Lessar sac(ans) Splenic vein 27. Hepatoduodenal ligament transmits Portal vein(ans) Pancreatic duct 28. In surgery hepatoduodenal ligament is injured which artery will be affected Hepatic artery (ans) Cystic artery 29.a man having echomosis.. able to stand on toes but have pain Planteris tendon rupture(ans)ref:dr mehtab notes in group Achiles tendon rupture 30.in hemolysis hb bind to Haptoglobin 31.in mastectomy which nerve is damaged Spinal accessory nerve Long throcic nerve(ans) 32. Most common congenital anomaly of head and neck Cystic hygroma Cleft plate(ans) 33.extraembyonic mesoderm derived from 105

Epiblast Hypoblast(ans) Endoderm 35.beta thalessemia scenrio 36.right eye swelling.... radiological study of orbit is normal Meningioma Glial gloma Retinal arteries obstruction 37.most commonly fructure of bone after 60 yrs Neck of femur (ans) Shaft of femur 36.parathyriod enlaged cells... Oxyphil , chief cells , both???? 37.ques abt ankle joint.. Deltoid ligament lies behind middle maleolar 39.visual area is supplied by Pca(ans) Mca 40. Mandibular branch of Trigeminal nerve passes through Foramen ovale(ans) Foramen rotandum 41.middle cerebral artry is branch of Internal carotid arted artery(ans) External carotid artey 42.type 3 reaction Arthus 43. Myelin form in cns Oligodendrocytes(ans) Schwan cells 44. Preganglionic fibers B fibers 45. Cardiac reserve( rabia ali mcq) 46.Most common Inheritance pattern A. Autosomal Dominant B. Autosomal Recessive C. X-linked recessive D. Multifactorial(ans) 47.necrosis in brain Liquefactive(ans) Coagulative 48.fructure humate bone Ulnar nerve(ans) Radial artery 49.antibodies in pernicious anemia Antiparietal antibodies (ans) Antityroxin antibodies Warm antibodies 50.collection of mix blood from Pulmonary artery(ans) Pulmonary vein Left atrium 51. Scienerio of achondroplasia 106

Autosomal Dominant (ans) X.linked recessive 52. ivu kidney more prominent .. Calyces (ans) pelvis pyramids 53.which of the following is responsible for maintaining body in shock Adreno sympathetic ( ans) ??? Cns ischemic response response 54.thyroid follicles that release hormone are derived frm.. Endoderm(ans) mesoderm 56.spinal cord ends at... L2, lower border of l1...(ans) 57.epidural space contains.... Venous plexues 58.most common dislocation of TMJ... Anterior posterior lateral... 59.sa node is pace maker bcux... Top of rt atrium(ans) Lower part of sulcus terminalis Something like in between conducting pathway 60.major supprt of utreus (ligament) Broad ligament Transverse cervical ligament (ans) 61.thyroid Drains to deep cervical lymp nodes 62.clostridium botulinum Flacid paralysis 63. slow growing tumor of thyroid Papillary carcinoma (ans) Medullary carcinoma 65. sensry loss on lower surface of deltoid musle and femur bone fructure nerve involved Axillary nerve(ans) Musculocutaneus 66. fractur of neck of humrus nerve damaged Axillary nerve 67. Regarding cephalic vein Arises from medial side of dorsal arch Lie between deltoid and pectoralis major muscle(ans) 68.nerve that supplies extensor of arm arises from.. Lateral cord medial cord posterior cord(ans) 69.a 60 yrs old man develops pain in calf muscles aftr walking 100 yards pain is due to... Popliteal (ans) posterior tibial 70. Ulcer 2nd part of duodenum which arty will u ligate 107

Gastroduodenal artery(ans) Right gastric artery 71.10 yrx boy repeated visits to opd with pallor frontal bossy thalasemia major(ans) Thalasemia minor Sickle cell disease 72.renal vein lies posterior to Renal artery Pancreas Aorta Sma(ans) 73.vagina and utrus epithelium frm Endoderm Mesoderm Endoderm and mesoderm 74.parathyriod enlarged cells... Oxyphil chief cells both 78.lumber triangle anteriorly bound by posterior border of Extrnal oblique quadratis lumbrm Serratus anterior(ans) 79. Regarding sarcoma Increased vascularity 80.icam nd vcam Leukocyte adhesion 81.Mononucleosis. Stucture most likely to be damaged even on minor trauma A. Liver B. spleen(ans) C.Gall bladder D. Pancreas 82.Barium swallow Feathery appearance in left abdomen A. Jejunum(ans) B. Ileum C. Stomach 83.Damage to middle cranial fossa. No lacrimation. damage to A. Ciliary ganglion (ans) B. Greater petrosal nerve 84.In spermiogenesis acrosomes are synthesized and secreted by A. Lysosomes B. Mitochondria C. Golgi complex (ans) D. Endoplasmic reticulum E. Peroxisomes 85.Which of the following is not isotonic to plasma A. 0.9% N/S B. 5% D/w(ans) C. Plasma protein fraction D. Ranger lactate E. 1.2% sodium bicarb 108

86.Highest Pco2 in A. Expired air B. Alveoli C. Pulmonary artery(ans)reference guyton D. Pulmonary vein 87.pregnant lady Jaundice Stone in bile duct. most important investigation A. Ggt(ans) B. Alkaline phosphatase C. Alt88 D. Ast 88.Somatostatin acts on beta islet cells A. Apocrine B. Holocrine C. Paracrine(ans) D. Neuroendocrine E. Endocrine 89.Glucocoticoids inhibit A. Corticotropin B. insulin 90.Most common Inheritance pattern A. Autosomal Dominant B. Autosomal Recessive C. X-linked recessive D. Multifactorial 91.Exocelomic membrane derived from? A. Somatopleuric mesoderm B. Splancnopleuric mesoderm C. Hypoblast D. Epiblast 92.Very small particles in respiratory tract are cleared by A. Ciliary transport B. Phagocytosis C. Expectoration D. Mass movement 93.Jaundice Stone in bile duct. most important investigation A. Ggt B. Alkaline phosphatase C. Alt D. Ast 94.External urethral sphincter related to which part of prostate A. Lateral B. Posterior C. Base D. Apex 95.Barium swallow Feathery appearance in left abdomen A. Jejunum B. Ileum C. Stomach 96.Trauma to head. No lacrimation. damage to A. Ciliary ganglion B. Greater petrosal nerve 109

C. Cervical sympathetic don't remember exactly 97.Parathyroid hyperplasia involves A. Oxyntic cell B. Chief cells C. Oxyntic + chief 98.Adreno cortical insufficiency cuases A. Alkalosis B. Hyponatremia C. Hypokalemia D. Hyperglycemia 99.Most common TMJ dislocation A. Anterior B. Posterior C. Inferior D. Lateral 100.Unilateral painless bulging of eye. blood profile normal. Radiologic study normal A. Meningioma B. Glioma of optic nerve C. Graves D. Mets E. Thyroid ophthalmopathy 101.Which of the following is not isotonic to plasma A. 0.9% N/S B. 5% D/w C. Plasma protein fraction D. Ranger lactate E. 1.2% sodium bicarb 102.Phospholipase most likely to be inhibited by A. Steroids B. Aspirin C. Streptokinase 103.Eparterial bronchus is related to A. Right superior B. Left superior C. Right middle D. Right inferior E. Left inferior 104.First observable sign of irreversible cell injury in cardiac cell A. Clumping of nuclear chromatin B. Hydropic change C. Cell Swelling D. Contraction bands in cytoplasm 105.Lumbar triangle bounded anteriorly by A. External abdominal oblique B. Litissimus dorsi 106.Anterior relation of left kidney directly without intervening peritoneum A. Spleen B. Duodenum C.Pancreas 110

D. Stomach E. Jejunum 107.Uterus and vaginal epithelium derived from A. Endoderm B. Mesoderm C. Endoderm + Mesoderm D. Mesoderm + Ectoderm 108.Major support of uterus A. Transverse cervical ligament B. Uterosacral Ligament C. Pelvic diaphragm D. Urogenital diaphragm 109.IVU of kidney done. Most prominent part would be A. Calyces B. Pyramid C. Medulla D. Cortex 110.Somatostatin acts on beta islet cells A. Apocrine B. Holocrine C. Paracrine D. Neuroendocrine E. Endocrine 111.Anterior to left renal vein A. Inferior mesenteric vein B. Aorta C. Renal artery D. Pancreas 112.Most common head and neck congenital anomaly A. Cleft lip and palate B. Preauricular cyst C. Problems of ear D. Cystic hygroma 113.Most important response as a whole A. Sympathetic/adrenocortical stimulation B. Brain Ischemic response 114.Highest Pco2 in A. Expired air B. Alveoli C. Pulmonary artery D. Pulmonary vein 115.Postpartum hemorrhage caused ARF. structure most likely damaged is A. Pct B. Dct C. Ascending loop of henle D. Descending loop of henle E. Collecting duct 116.In spermiogenesis acrosomal reaction occurs by A. Lysosomes B. Mitochondria C. Golgi complex 111

D. Endoplasmic reticulum E. Peroxisomes 117.Fracture of hamate can damge? A. Radial artery B. Radial nerve C. Ulnar nerve D. Median nerve E. Flexor carpi ulnaris 118.Detrusor muscle is supplied by A. Pudendal (S2-4) B. Parasympathetic (S2-4) C. Inferior hypogastric plexus 119.Axons of vestibular nuclear complex ends at A. Dentate B. Granular layer C. Cochlear plz add options 120.Posterior most in the posterior limb of internal capsule A. Optic radiations B. Corticospinal tract of upper limb C. Corticospinal tract of lower limb 121.Pain in left calf on walking relieved by rest. Involves A. Popliteal artery B. Post tibial C. Ant tibial D. Profunda femoris E. Femoral 122.Mononucleosis. Stucture most likely to be damaged even on minor trauma A. Liver B. spleen C.Gall bladder D. Pancreas

= - = - = - = - = - = - = - = - = - = - = - = - = - = - = - = - = - = - = - = - = - = - = - = - = - = - =

112

(10.) Community medicine 7th March 2016 (Morning) by Qaisar Javed (90+85) Paper-1 (90 )MCQS

1. True about Glycocalyx - fuzz like coat on external surface??/ carrier protein/ lipid mem

2. Hypermagnesemia - Decrease release of Ach/ hyperreflexia?

3. Brain regulated by - Pco2

4. Confidentiality breach - if pt auth you

5. H202 in which organelle - peroxisomes

6. Cannot regenerate- Lens

7. True statement -osteoblasts/clast are multinucleated or Uncalcified organic matrix is called osteoid - Question/Stems not properly recalled..

8. 2nd arch derivative- Stylohyoid

9. Liquefactive necrosis - in brain

10.Tfr of radicals(atoms) from one gp to another- oxidoreductases

11.Caculate median - 27.5 Answer

12.Golgi muscle tendon - tension /relaxation info ??

13.ILD non caseating granuloma with hilar nodes- Sarcoidosis

14.RBC antigen - Glycoprotein / Glycolipid ?

15.Most common genetic disorder - Multifactorial

16.Muscle of fright and Sadness- Platysma

17.Throid lymph Drainage- deep cervical lymph nodes

18.Temporal artey realted - Auriculotemporal nerve

19.Acute Renal failure part affected - Pct?? / loop of henle

20.Irreversible cell injury in myocardium- Contraction bands in cytoplasm?? / Nuclear chromatin appearance

21.DVT emboli - Pulm artery?

22.Pco2 max in - Pulm Vein/ pulm artery ??/ expired air

23.Shoulder pain referred from pleura - C345

24.Lt suprarenal drain - Lt renal vein 113

25.1/3rd body water - ECF

26.Councilman bodies seen - Apoptosis

27.Basic drug binds - alphaglycoprotein

28.Case control typical scenario - disease and not diseased

29.ICAM and VCAM - adhesion molecules

30.Symphysis pubis joint - Secondary cartilaginous joint

31.Pyogenic peritonitis - Bacteroids

32.Diff Malignant and Benign tumor - Invasion? / metastasis

33.Microscopic feature Malignant tumor - invasion

34.Metaplasia - epithelium change

35.Goblet cells covert to Clara- Terminal bronchioles

36.No lymph nodules - Thymus (spleen/lymph nodes..)

37.Megalobalstic anemia invest - Intrinsic factor antibodies

38.Scenario Achondroplasia - Autosomal dominant

39.Pt lying on bed at room temp - Conduction

40.Middle meningeal artery - Foramina Spinosum

41.Incr PT and PTTK - Vit K def ?

42.Wound strength - Type III collagen

43.Decrease Gentamicin clearance in old age - Decrease renal func

44.Lower limit of subarachnoid- s2 (L2 / ..)

45.Above hilum lt lung - Arch of aorta

46.Congenital cataract- Rubella

47.Virus act by - altering proteins synthesis

48.No valves seen in - SVC (IVC/..)

49.Pt support in our culture - family

50.Fast pain fibre- A delta 114

51.Preganglionic nerve fiber- Beta fiber?

52.Pleural drainage- area of max dullness at upper border of lower rib

53.TB Dx - sputum AFB

54.Extraembryonic coelomic membrane - endoderm

55.Somatostain acts on beta cells - paracrine/holocrine/autocrine/endocrine/neuroendocrine

56.Not isotonic to plasma - 1.2% HCO3 ?

57.Ventricular filling - 3rd heart sound

58.Glucorticoids inhibit - Corticotrophin?

59.In spermiogenesis Acrosome - Lysosome/ Golgi

60.Organ which utilizes glucose - brain

61.pt came at 2am with chest pain - CKMB (No other best options were there)

62.Lower motor neuron lesion - flaccid paralysis

63.CNS myelination - Oligodendrocytes

64.Receptor action via Phospholipase C - Vasopressin induced smooth muscle contraction

65.Flight or fright Stimulation- Decrease release of bronchial secretions ( pupillary constriction/Incr GI motility)

66.Atrial fibrillation - Pulse deficit

67.Hot sunny day, sudden collapse - Excessive sweating

68.C1 vertebra feature - lacks vertebral body

69.Pulse pressure max at - renal artery/ pulm artery/...?

70.Dec plasma proteins - Inc GFR

71.Cardiac reserve- Max % by which CO can be inc above normal

72.Osmosis depends?? - no of partices ?

73.1 litre blood loss late to adjust ?? - rbc mass/ renin/ aldosterone

74.Incr TPR ?? - ??

75.5% dextran in large amount given - adh inhibition?

76.Plasma mem - 100nm/ extent to size of membrane?? 115

77.Cardiac pleural serous mem supplied by ?? - cardiophrenic/vagus

78.Betel nut -submucosal fibrosis

79.In preg jaundiced Inv performed - GGT

80.TB antibodies are - Cell bound

81.Regarding ?? - Hiv dec helper cells ?

82.Mot imp response as a whole - CNS ischemic response / sympathetic adrenal response

83.Corticosteroids anti inflammatory action by - ??

84.In Iron def anemia in pregnacy, Increased - TIBC

85.Very small particles in respt ract cleared by - ciliary / phagocytosis

86.APKD assoc - renal failure/ cerebral hrmorrhage

87.Acute glomerunephritis proteinuria- Basement mem defect / subepithelial deposit/IgG dep Basement mem

88.Organelle having nucleus ? - Mitochondria

89.Motor unit - muscle fascicle innervated by single motor nerve fibre

90.Vagus nerve true? - passes thru middle of jugular foramin / beneath subclavian artry in neck

7th March 2016 (Morning Shift) Community Medicine Paper 2

1.Vit A dose 2 yrs age, prevention - IU

2.Riboflavin def .. staple diet.. rice?

3.Another vit def assoc with Niacin?

4.Vit A highest in ? Potato> green leafy veg

5.Scenarios related to persuation

6.motivation

7.Counselling

8.propaganda

9.Symposium

10.cause specific fatality rate scenario 116

11.Incidence formula - denominator

12.rabbit disease- tularemia

13.echinococuus granulosis- transmission dogs? sheep cattle fish

14-15.cas control scenario-- 2-3 scenarios

16-17.cohort scenario 2-3 scenarios

18.positively skewed curve - Mean>Mean>Mode

19.pinworm Rx

20.Human dev index composition

21.community participation scenario

22.Integrated health realted scenario

23.Secondary attk rate numerical

24.case fatality rate numerical

25.OCD Scenario- Personality disorder

26.Scenario- lack of awareness among community

27.MCHC succesful... comm participation

28.Primordial prevention

29.disability- tertiary prev

30.Use of helmet for motorcycle- Prim prev

31.Impairment Definition

32.neonatal mort rate- shows late fetal deaths

33.snowstorm apperance- silicosis

34.Improving working environment --- scenario

35.DM is inc in dev countries -- different suboptions

36.Vasculotoxic Russel viper? common krait

37.p value .. alpha error definition

38.pt not treated by local dr but sent to tertiary care for atls mngmnt.. First lvl referral .. other options prim /sec/ ter prev/first lvl care 117

39.Lead time - def

40.convalescent carrier?? Definition

41.min percentage of essen fats for CHD patient- 10% of energy intake

42.mild/moderate arm circumference in anthropometric measurement

43.waterloo class for growth charts

44.PEM min protein lvl below % -- 7?

45.Obese pt , essential fats limited to - ?%

46.human milk contain protiens- 1-2 gm

47.Diphtheria immunized 2 yrs bk... again rx? Penicillin + Acitive or passive immunity

48.dewdrop lesion - chicken pox

49.mid arm circumference in children mild/moderate - 12.5-13.5cm

50.CHD and Cataract infection -- Rubella

51.TB mother .. child tuberculin positive -- rx ? BCG, BCG+ATT, INH for 3months, Ig

52. Amenorrhea and sterility --- caues?? lead, mercury , etc ?

53.rate - numerator in denominator

54.HTN & IHD in male and female --- confounding factor- female sex ?

55.sensitivity incr - specifity dec

56.Vit A def- Night blindness

57.female gathering , awareness on cancer common-- breast CA

58.non modifiable factor - cigarette smokingr

59.Alcohol and LDL assoc being studied lead to MI assoc -- this is called ?

60.Diabetes screening- urine test for glucose or BSR?

61.Viral Infec not causing congenital abnormality?? HIV, HSV, HZV, Rubella, CMV

62.IgE - hypersensitivity

63.igG - Most abundant

64.Targeted screening- workers of factory 118

65.Population doubling time for growth rate 3% - 20-25 yrs

66.Decreased population growth of country analysed by -- Inc mortalilty rate ?

67.Net reproduc rate - assoc with mortality rate

68.median in given calculations= 27.5 answer - past paper

69.Demographic Cycle- Pakistan entered 3RD STAGE

70.tetanus campaign by unicef = < 1 case per 1000

71.Worms in feces by eating pork - T.solium

72.Ozone depletion by factories- by CFC

73.Green house effect mainly by- CO2

74.Air pollution- Soiling index

75.Strength of sewage - Biochemical O2 demand

76.ILD with non caseating granuloma and hilar lymp nodes- Sarcoidosis

77.LBW < 2.5 kg weight

78.Smoking in pregnancy-- IUGR

79.Wheel of causation - main component-- HOST

80.Fecal contamination recent - e.Coli?

81.Adding saccharide in ice cream - Food additive

82.Critical Path method - Network analysis

83. Snake bite – Immobilze pt

84. Teaching Ors prep to mothers – Demonstration

85. Stratified random sampling

=-=-=-=-=-=-=-=-=-=-=-=-=-=-=-=-=-=-=-=-=-=-=-=-=-=-=-=-=-=-=-=-=-=-=-=-=-=-=-=-=-=- The End- Compiled by : Amlodipine Besylate =-=-=-=-=-=-=-=-=-=-=-=-=-=-=-=-=-=-=-=-=-=-=-=-=-=-=-=-=-=-=-=-=-=-=-=-=-=-=-=-=-=-